Class Xi Chemistry Study Materialterm-1 Xi

You might also like

Download as pdf or txt
Download as pdf or txt
You are on page 1of 103

STUDENT SUPPORT MATERIAL

1
CHIEF PATRON
Dr. JAIDEEP DAS,
DEPUTY COMMISSIONER,
KVS, AHMEDABAD REGION
PATRON
SMT. SHRUTI BHARGAVA,
ASSISTANT COMMISSIONER,
KVS, AHMEDABAD REGION
UNDER THE SUPERVISION OF
SHRI MANISH JAIN,
PRINCIPAL,
KENDRIYA VIDYALAYA ONGC
ANKHLESHWAR

CO–ORDINATION TEAM
PHYSICAL CHEMISTRY ORGANIC CHEMISTRY INORGANIC CHEMISTRY
SH. P K PATIDAR, DR. M R CHOUDHARY, K MRS. SABIHA SHAIKH,
KV ONGC CHANDKHEDA V SAC K V No.1
AHMEDABAD SURAT

REVIEW COMMITTEE

PHYSICAL ORGANIC INORGANIC QUESTION


CHEMISTRY CHEMISTRY CHEMISTRY
PAPER

SH. BHARAT KUMAR SH. S K SONI, SH. ATUL TIWARI, SH. INDRAMAL,
PANDYA, K.V NO. 3 KV AHMEDABAD CANTT KV 2 EME BARODA K V INS VALSURA
SURAT

2
CONTENT DEVELOPEMENT TEAM

CHAPTERS NAME NAME OF TEACHER NAME OF SCHOOL

1 SOME BASIC CONCEPTS OF


MR. YOGESH MEHARDA KV–PORBANDAR
CHEMISTRY
2 STRUCTURE OF ATOM MR. DEEPAK NO.3 AFS JAMNAGAR

3 CLASSIFICATION OF ELEMENTS
MR. BHANWAR SINGH
AND PERIODICITY IN KV ONGC BARODA
RATHORE
PROPERTIES
4 CHEMICAL BONDING AND MR. MUKESH KUMAR
K V RAJKOT
MOLECULAR STRUCTURE SOLANKI
5 REDOX REACTIONS MR. VIKASH KUMAR
KV DIU
BISNOI
6 HYDROGEN KV ONGC
MRS. UMA UMMINIDI
CHANDKHEDA
7 ORGANIC CHEMISTRY: SOME
BASIC PRINCIPLES AND MR. A K SAKERWAL KV AFS SAMANA
TECHNIQUES
8 MODEL QUESTION PAPER K V SEC– 30
MR A K JHA
GANDHINAGAR

3
CHEMISTRY
CLASS–XI
INDEX

S NO CONTENT PAGE NO

1 SYLLABUS THEORY TERM–1 & MARKING SCHEME 6–7

2 SYLLABUS PRACTICAL TERM–1 & MARKING SCHEME 8

3 SOME BASIC CONCEPTS OF CHEMISTRY 09–20

4 STRUCTURE OF ATOM 21–30

5 CLASSIFICATION OF ELEMENTS AND PERIODICITY 31–41


IN PROPERTIES
6 CHEMICAL BONDING AND MOLECULAR STRUCTURE 42–52

7 REDOX REACTIONS 53–63

8 HYDROGEN 64–72

9 ORGANIC CHEMISTRY: SOME BASIC PRINCIPLES 73–88


AND TECHNIQUES
10 MODEL QUESTION PAPER 89–98

11 MARKING SCHEME 99

SYLLABUS FOR SESSION–2021–22


CLASS XI Term–I
SN UNIT Periods Marks
1 SomeBasicConceptsofChemistry 10 11

2 StructureofAtom 12
3 ClassificationofElementsandPeriodicityinProperties 6
4
4 ChemicalBondingandMolecularStructure 14
6
5 RedoxReactions 4
6 Hydrogen 4 5
7 OrganicChemistry:SomebasicPrinciplesandTechniques 10 9
TOTAL 60 35
4
Some Basic Concepts of Chemistry:
General Introduction: Importance and scope of Chemistry. Atomic and
molecular masses, mole concept and molar mass, percentage composition,
empirical and molecular formula, chemical reactions, stoichiometry and
calculations based on stoichiometry.

Structure of Atom:
Bohr's model and its limitations, concept of shells and subshells, dual nature
ofmatter and light, de Broglie's relationship, Heisenberg uncertainty principle,
concept of orbitals,quantum numbers, shapes of s, p and d orbitals, rules for
filling electrons in orbitals – Aufbauprinciple, Pauli's exclusion principle and
Hund's rule, electronic configuration of atoms, stability of half–filled and
completely filled orbitals

Classification of Elements and Periodicity in Properties:


Modern periodic law and the present form of periodic table, periodic trends in
properties of elements –atomic radii, ionic radii, inert gas radii, Ionization
enthalpy,electron gain enthalpy, electronegativity, valency. Nomenclature of
elements with atomic number greater than 100.

Chemical Bonding and Molecular Structure:


Valence electrons, ionic bond, covalent bond, bond parameters, Lewis
structure, polar character ofcovalent bond, covalent character of ionic bond,
valence bond theory, resonance, geometry of covalent molecules, VSEPR
theory, concept of hybridization, involvings, p and d orbitals and shapes of
some simple molecules, molecular orbital theory of homonuclear diatomic
molecules (qualitative idea only), Hydrogenbond.

Redox Reactions:
Concept of oxidation and reduction, redox reactions, oxidation number,
balancing redox reactions, in terms of loss and gain of electrons and change in
oxidation number.

Hydrogen:
Position of hydrogen in periodic table, occurrence, isotopes, hydrides–ionic
covalent and interstitial; physical and chemical properties of water, heavy water,
hydrogen as afuel

Organic Chemistry: Some basic Principles and Techniques:


General introduction, classification and IUPAC nomenclature of organic
compounds. Electronic displacements in a covalent bond: inductive effect,
electromeric effect, resonance and hyper conjugation. Homolytic and
heterolytic fission of a covalent bond: free radicals, carbocations, carbanions,
electrophiles and nucleophiles, types of organic reactions.

********************************************
5
PRACTICALS
Term I:

A 15– mark Practical would be conducted under the supervision of


subject teacher. This would contribute to the overall practical marks for
the subject.
OR
In case the situation of lockdown continues until Nov–Dec 2021, a Practical
Based Assessment (pen– paper) of 15 marks would be conducted at the
end of Term I.
EvaluationScheme

S.No Practical Marks


1. Volumetric Analysis 8
2. Content Based experiment 2
3. Class record and viva (Internal Examiner) 5
TOTAL 15

Micro–chemical methods are available for several of the practical


experiments, wherever possible such techniques should be used.

A.Basic Laboratory Techniques

1. Cutting glass tube and glass rod


2. Bending a glass tube
3. Drawing out a glass jet
4. Boring a cork

B. Characterization of Chemical Substances (2 Marks)

1. Determination of melting point of an organic compound.


2. Determination of boiling point of an organic compound.

C. Quantitative Estimation (8 marks)

i. Using a mechanical balance / electronic balance.


ii. Preparation of standard solution of Oxalic acid.
iii. Determination of strength of a given solution of Sodium hydroxide
by titrating it against standard solution of Oxalic acid.
iv. Preparation of standard solution of Sodium carbonate.
v. Determination of strength of a given solution of hydrochloric
acid by titrating it against standard Sodium Carbonate
solution.
SOME BASIC CONCEPTS OF CHEMISTRY

PART–1(Study based MCQ’s)

Passage 1.The concentration of a solution is expressed in many ways molarity,


molality, mole fraction, g/litre ,ppm , ppb, and % . The molarity is number of
moles of solute per litre of solution whereas molality is number of moles of
solute per kilogram of solvent. Mole fraction on the other hand is ratio of moles
of any component to the total moles of all components in the solution.
Answer the following questions:
Q1. Which concentration term will be affected by temperature?
(A) Molality
(B) Molarity
(C) Mole fraction
(D) Mass %

Q2. What is mole fraction of pure water


(A) 0.5
(B) 1
(C) 0.75
(D) Cannot be predicted

Q3. An aq. solution of urea contains 6.022 x1022 molecules of urea in its
100 ml. What is molarity of the solution? Molar mass of urea is 60 u.
(A) 1
(B) 2
(C) 0.25
(D) 0.1

Q4.What is unit of molarity?


(A) mol/kg
(B) mol/l
(C) g/l
(D) No unit

Q5. A solution of glucose in water is equimolar. What is mole fraction of


glucose?
(A) 0.1
(B) 1.0
(C) 0.25
(D) 0.5
Passage 2.Empirical formula is the simplest formula of the compound which
gives the atomic ratio of various elements present in one molecule of the
compound. However, the molecular formula of the compound gives the number
of atoms of various elements present in one molecule of the compound.
Molecular formula = (Empirical formula) x n

A compound may have same empirical and molecular formulae. Both these
formulae are calculated by using percentage composition of constituent
elements.
Answer the following questions:
Q1. Two metallic oxides contain 27.6% and 30% oxygen respectively.
If the formula of first oxide is M304, that of second will be:
(A) MO
(B) M02
(C) M205
(D) M203

Q2. Which of the following compounds have same molecular and


empirical formula?
(A) Formaldehyde
(B) Glucose
(C) Fructose
(D) Acetic acid

Q3. Which of the following represents the formula of a substance which


contains 50% oxygen?
(A) N20
(B) CO2
(C) NO2
(D) CH30H

Q4. An oxide of iodine (I = 127) contains 25.4 g of iodine and 8 g of


oxygen. Its formula could be:
(A) I203
(B) I20
(C) I205
(D) I207

Q5. In which compound the formula mass is used


(A) NaCl
(B) CH4
(C) CO2
(D) H2O
Passage 3. Mole, also spelled mol, in chemistry, a standard scientific unit for
measuring large quantities of very small entities such as atoms, molecules, or
other specified particles.
The mole designates an extremely large number of units, 6.02214076 × 10 23.
The General Conference on Weights and Measures defined the mole as this
number for the International System of Units (SI) effective from May 20, 2019.
The mole was previously defined as the number of atoms determined
experimentally to be found in 12 grams of carbon–12. The number of units in a
mole also bears the name Avogadro’s number, or Avogadro’s constant, in honor
of the Italian physicist Amedeo Avogadro (1776–1856). Avogadro proposed that
equal volumes of gases under the same conditions contain the same number of
molecules, a hypothesis that proved useful in determining atomic and molecular
weights and which led to the concept of the mole.

Answer the following questions.


Q1. How many moles of Hydrogen atoms are present in 4 moles of H2?
(A) 2
(B) 4
(C) 8
(D) 12

Q2. Number of molecules of O2 present in 36 grams of H2O


(A) 3.01 × 1023
(B) 9.03 × 1023
(C) 12.04 × 1023
(D) 6.02 × 1023

Q3. Which one will have maximum number of atoms?


(A) 4 grams He
(B) 4 moles of He
(C) 8 grams He
(D) 2 moles of He

Q4. The molarity of pure water is


(A)18 mole/liter
(B)5.56 mole/liter
(C)55.5 mole/liter
(D)7 mole/liter

Q5. If one spends 6.02 × 1013 rupees every second then how many
seconds are required to spend 1 mole rupees?
(A) 1023s
(B) 1024s
(C) 1010s
(D) 108s
PART–2 (Study based A–R questions)

Passage 1: The reagent that in a chemical reaction gets over and stops the
reaction is called as limiting reagent. The amount of reactants used and product
formed is dependent on the amount of limiting reagent.

In the following questions, a statement of Assertion (A) followed by a


statement of Reason (R) is given. Choose the correct option out of the choices
given below each question.
(A) Both A and R are true and R is the correct explanation of A.
(B) A is true but R is false.
(C) A is false but R is true.
(D) Both A and R are false.
Q1. Assertion: When 12 g carbon reacts with 30 g of oxygen it will not form 42
g of CO2
Reason:: O2 in above reaction is limiting reagent.

Q2. Assertion:When 1 g of hydrogen reacts with 8 g of oxygen 9 g of water


will form.
Reason:H2 is limiting reagent in this reaction.

Q3. Assertion:The reagent that is left over in the reaction is called as reagent
in excess.
Reason:Reagent in excess decides how much of product will form.

Q4. Assertion:10 moles of carbon monoxide will react with 10 moles of O2


completely.
Reason:CO in the above reaction is limiting reagent.

Q5. Assertion:100 g CaCO3 and 2 litres of one molar HCl solution will produce
11.2 L CO2 at STP.
Reason:HCl will behave as limiting reagent in the above reaction.

Passage–2: The concentration of a solution or the amount of substance


present in its given volume can be expressed in any of the following ways.

In the following questions, a statement of Assertion (A) followed by a


statement of Reason (R) is given. Choose the correct option out of the choices
given below each question.
(A) Both A and R are true and R is the correct explanation of A.
(B) A is true but R is false.
(C) A is false but R is true.
(D) Both A and R are false.
Q1. Assertion: If in a binary solution mole fraction of one component is 0.5
then the mole fraction of the other component will also be 0.5.
Reason:The sum of mole fraction is equal to one.

Q2. Assertion: Molarity gets affected by temperature.


Reason:Molarity is a volume–based quantity which gets affected by
temperature

Q3. Assertion: On dilution molarity increases.


Reason: Molarity is the ratio of number of moles of solute and volume of
solution in litres.

Q4. Assertion: Molality is preferred over molarity.


Reason: On dilution number of moles of solute never changes.

Q5. Assertion: Molarity of pure water is 55.56 M.


Reason: Molarity is temperature dependent parameter.

PART–3 (Assertion–Reason questions)

In the following questions, a statement of Assertion (A) followed by a


statement of Reason (R) is given. Choose the correct option out of the choices
given below each question.
(A) Both A and R are true and R is the correct explanation of A.
(B) A is true but R is false.
(C) A is false but R is true.
(D) Both A and R are false.

Q1. Assertion: The empirical mass of ethene is half of its molecular mass.
Reason: The empirical formula represents the simplest whole–numberratio of
various atoms present in a compound.

Q2. Assertion: One atomic mass unit is defined as one twelfth of the mass of
one carbon–12 atom.
Reason: Carbon–12 isotope is the most abundant isotope of carbon
and has been chosen as standard.

Q3. Assertion: Significant figures for 0.200 is 3 whereas for 200 it is 1.


Reason: Zero at the end or right of a number are significant provided
they are not on the right side of the decimal point.

Q4. Assertion: Combustion of 16 g of methane gives 18 g of water.


Reason: In the combustion of methane, oxygen is one of the products.

Q5. Assertion: 1.231 has 3 significant figures


Reason: All numbers right to the decimal point are significant.

Q6. Assertion: The number of O atoms in 16 g of oxygen and 16 g of ozone is


same.
Reason: Each of the species represent 1 g–atom of oxygen
Q7. Assertion: Atomic mass of Na is 23.
Reason: An atom of sodium is 23 times heavier than 1/12th mass of one atom
of C–12 isotope.

Q8. Assertion: The number of significant figures in 507000 is three.


Reason: In 507000, not all the zeros are significant.

Q9. Assertion: Avogadro's number is a dimensionless quantity.


Reason: It is a number of atoms or molecules in one–gram mole.

Q10. Assertion: Molarity of pure water is 55.5 M.


Reason: Molarity is temperature dependent parameter.

PART–4 (MCQ’s)

Q1. What is the mass percentage of carbon in carbon dioxide?


(A) 0.034%
(B) 27.27%
(C) 3.4%
(D) 28.7%
Q2. The empirical formula and molecular mass of a compound are CH2O
and 180 g respectively. What will be the molecular formula of the
compound?
(A) C9H18O9
(B) CH2O
(C) C6H12O6
(D) C2H4O2
Q3. What will be the molarity of a solution, which contains 5.85 g of
NaCl (MM=58.44 g/mole) per 500 mL?
(A) 4 mol L–1
(B) 20 mol L–1
(C) 0.2 mol L–1
(D) 2 mol L–1
Q4. The number of atoms present in one mole of an element is equal to
Avogadro number. Which of the following element contains the greatest
number of atoms?
(A) 4 g He
(B) 46 g Na
(C) 0.40 gCa
(D) 12 g He
Q5. The number of significant figures in 6.02×1023 are
(A) 23
(B) 3
(C) 4
(D) 26
Q6. How many moles of He gas occupy 22.4 L at 0˚C at 1 atm pressure
(A) 0.11
(B) 0.90
(C) 1.0
(D) 1.1
Q7. 0.5 mole of triatomic gas contains ____________________atoms
(A) 6.02×1023
(B) 3.01×1023
(C) 9.03×1023
(D) 2.03×1023

Q8. 6.02 ×1020 molecules of ammonia (NH3) are present in 100 mL of


its solution. The Molarity of the solution is
(A) 0.1
(B) 0.01
(C) 1.0
(D) 0.001

Q9. 1 Picometre is equal to


(A) 10–10 m
(B) 10–15 m
(C) 10–9 m
(D) 10–12 m

Q10. Which of the following is dependent of temperature?


(A) Molarity
(B) Molality
(C) Mole fraction
(D) Mass percentage

Q11. The molarity of a solution obtained by mixing 750 mL of 0.5 M HCl


with 250 ml of 2 M HCl will be
(A) 0.975 M
(B) 0.875 M
(C) 1.0 M
(D) 1.175 M

Q12. A mixture having 2 g of H2 and 32 g of oxygen occupies a volume


of__ at NTP.
(A) 22.4 L
(B) 44.8 L
(C) 11.2 L
(D) 67.2 L

Q13. Which measurement is more precise?


(A) 4
(B) 4.0
(C) 4.00
(D) None of the above
Q14. In a reaction A + B2→ AB2if 2 moles of A and 3 moles of B2 are
used then which one will be a limiting reagent
(A) no limiting reagent
(B) B2
(C) A
(D) Data is insufficient
Q15. Express 0.00016 in the scientific notation
(A)1.6 × 10–4
(B) 1.6 × 10–3
(C) 1.6 × 10–5
(D) 1.6 × 10–2

Q16. The amount of water produced by the combustion of 32 gram of


methane CH4 + 2O2→ CO2 + 2H2O
(A) 30 g
(B) 36 g
(C) 72 g
(D) 70 g

Q17. SI unit for the amount of substance is


(A) Kilogram
(B) Gram
(C) Candela
(D) Mole

Q18. 10 mol of Zn react with 10 mol of HCl. Calculate the number of


moles of H2 produced– Zn + 2HCl → ZnCl2 + H2
(A) 5 Mole
(B) 10 Mole
(C) 20 Mole
(D) 2.5 mole

Q19. Which of the following terms is unitless?


(A) Molality
(B) Molarity
(C) Mole fraction
(D) Mass percent

Q20. One nm is equal to:


(A) 10–9 m
(B) 10–15 m
(C) 10–10 m
(D) 10–12 m

Q21. Given the numbers: 161cm, 0.161cm, 0.0161 cm. The number
of significant figures for the three numbers are
(A) 3,4 and 5
(B) 3,3 and 3
(C) 3,3 and 4
(D) 3,4 and 4

Q22. 350C in Fahrenheit will be


(A) 90
(B) 91
(C) 95
(D) 99
Q23. Addition of 6.65 × 104 and 8.95 × 103 is
(A) 7.545 × 10–4
(B) 7.545 × 108
(C) 7.545 × 1016
(D) 7.545 × 104

Q24. Multiplication of 2.5×1.25 will be


(A) 3.1
(B) 3.12
(C) 3.125
(D) 3.1254

Q25. If 500 mL of a 5M solution is diluted to 1500 mL, what will be the


molarity of the solution obtained?
(A) 1.5 M
(B) 1.66 M
(C) 0.017 M
(D) 1.59 M

Q26. What is the mass percentage of carbon in carbon dioxide?


(A) 0.034 %
(B) 27.27 %
(C) 3.4 %
(D) 28.4 %

Q27. One mole of any substance contains 6.022 × 1023 atoms or molecules.
Number of molecules of H2SO4 present in 100 mL of 0.02M aq. H2SO4 solution is
(A) 12.044 × 1020 molecules
(B) 6.022 × 1023 molecules
(C) 1 × 1023 molecules
(D) 12.044 × 1023 molecules

Q28. How many significant figures should be present in the answer of 5 × 5.364
(A) 1
(B) 2
(C) 3
(D) 4
Q29. Given that the abundances of isotopes 54Fe, 56
Fe and 57
Fe are 5%,90%
and 5% respectively, the atomic mass of Fe is:
(A) 55.85 µ
(B) 55.95 µ
(C) 55.75 µ
(D) 56.05 µ
Q30. 4.6 x 1022 atoms of an element weigh 13.8 g. The atomic mass of the
element is:
(A) 290
(B) 180
(C) 34.4
(D) 10.4
Answer–key
PART–1

Passage 1 Passage 2 Passage 3


Question Answer Question Answer Question Answer
number number number
1 B 1 D 1 C
2 B 2 A 2 D
3 A 3 D 3 B
4 B 4 C 4 C
5 D 5 A 5 C
PART–2
Passage 1 Passage 2
Question Answer Question Answer
number number
1 A 1 A
2 B 2 A
3 B 3 C
4 C 4 B
5 D 5 B
PART–3
Assertion and Reason
Question number Answer Question number Answer
1 A 6 A
2 C 7 A
3 C 8 D
4 D 9 A
5 D 10 B
PART–4
MCQ’s
Question number Answer Question number Answer
1 B 16 C
2 C 17 D
3 C 18 A
4 D 19 C
5 B 20 A
6 C 21 B
7 C 22 C
8 B 23 D
9 D 24 A
10 A 25 B
11 B 26 B
12 B 27 A
13 C 28 A
14 C 29 B
15 A 30 B
UNIT–2 STRUCTURE OF ATOM

Structure of Atom: Bohr's model and its limitations, concept of


shells and subshells, dual nature of matter and light, de Broglie's
relationship, Heisenberg uncertainty principle, concept of
orbitals, quantum numbers, shapes of s, p and d orbitals, rules for
filling electrons in orbitals – Aufbau principle, Pauli's exclusion
principle and Hund's rule, electronic configuration of atoms,
stability of half–filled and completely filled orbitals.
–––––––––––––––––––––––––––
CASE STUDY BASED (MCQ)
CS–1
Electrons revolve around the nucleus only in stationarypath, called energy levels
or orbits or shells. Each energylevel has a definite energy associated with it. As
one movesaway from the nucleus, the energy of the statesincreases.According
to Bohr‟s theory, when an electron jumps from ground state to excited state, it
emits a radiation of definite frequency (or wavelength). Corresponding to the
wavelength of each photon of light emitted, a bright line appears in the
spectrum.
1.According to Bohr‟s theory, the angular momentum of an electron in 5th orbit
is– (A) 10h/ π
(B) 2.5 h/ π
(C) 25h/ π
(D)1.0h/ π
2. Which of the following statements do not form a part of Bohr‟s model of
hydrogen atom ?
(A) Energy of the electrons in the orbits are quantized
(B) The electron in the orbit nearest the nucleus has the lowest energy
(C) Electrons revolve in different orbits around the nucleus
(D) The position and velocity of the electrons in the orbit cannot be determined
simultaneously.
3. The orbitals are called degenerate when–
(A) they have the same wave functions
(B) they have the same wave functions but different energies
(C) they have different wave functions but same energy
(D) they have the same energy
4. The spectrum of He is expected to be similar to that
(A) H
(B) Li+1
(C) Na
(D) He
5. According to the Bohr theory, which of the following transitions in the
hydrogen atom will give rise to the least energetic photon?
(A) n = 6 to n = 1
(B) n = 5 to n = 4
(C) n = 6 to n = 5
(D) n = 5 to n = 3
CS–2
Concept of movement of an electron in an orbit was replaced by the concept of
probability of finding electron in an orbital due to de–Broglie concept of dual
nature of electron and Heisenberg uncertainty principle.According to de–Broglie,
moving particles behave like wave and matter both i.e. show dual behavior.
6. For given energy, E = 3.03 × 10–19 joule corresponding wavelength is–
(h = 6.626 × 10–34Jsec, c = 3 × 108 m/sec)
(A) 65.6 nm
(B) 6.56 nm
(C) 3.4 nm
(D) 656 nm
7. de–Broglie‟s equation is–
(A) λ=h/p
(B) λ = h/2p
(C) λ = p/h
(D) λ = h/v
8. de–Broglie‟s equation is applicable in case of–
(A) microscopic particles
(B) macroscopic particles
(C) for both a and b
(D) none of the above
9. The de–Broglie wavelength of a tennis ball of mass 60g moving with a
velocity of 10m/s is approximately–
(A) 10–25 m
(B) 10–33 m
(C) 10–31 m
(D) 10–16 m
10. The angular momentum of an electron is zero. In which orbital may it be
present–
(A) 2s
(B) 2p
(C) 3d
(D) 4f

CS–3
An orbital is the three dimensional space around the nucleus within which the
probability of finding an electron is maximum. The maximum number of
electrons present in any orbital is two. An orbit is a welldefined circular path
around the nucleus in which the electron revolves.The maximum number of
electrons in any orbit is given by 2n2, where n is the number of the orbit.

11. The Bohr orbit radius for the hydrogen atom (n = 1) is approximately
0.530 A0. The radius for the first excited state (n = 2) orbit is (in A0)
(A) 4.77
(B) 1.06
(C) 0.13
(D) 2.12
12. Maximum number of electrons in a subshell of an atom is determined by the
following
(A) 2l + 1
(B) 4l – 2
(C) 2n2
(D) 4l + 2
13. The number of spherical nodes in 3p orbitals are
(A) one
(B) three
(C) two
(D) None of these
14. If r is the radius of the first Bohr‟s orbit, the radius of nth orbit of H–atom is
given by
(A) rn2
(B) rn
(C) r/n
(D) r2 n2
15. The maximum number of electrons present in any orbital is –
(A) 2
(B) 6
(C) 10
(D) 14

CASE STUDY BASED (A–R)


In the following questions (Q.16 to Q.25) a statement of Assertion
followed by a statement of Reason is given. Choose the correct answer
out of the following choice.
(A) Assertion and Reason both are correct statements but Reason is
correct explanation for Assertion.
(B)Assertion and Reason both are correct statements but Reason is
not correct explanation for Assertion.
(C)Assertion is correct statements but Reason is wrong statement.
(D)Assertion is wrong statement but Reason is correct.

CS–4

When a set of equivalent orbitals (degenerate orbitals) is either fully


filled or half–filled, i.e. each containing one or a pair of electrons, the
atom gain more stability. It is well known that symmetry leads to
stability. The completely filled or half filled subshells have symmetrical
distribution of electrons in them and are therefore more stable. This
effect is more dominant in d and f–sub–shells.
16. A– p–orbital is dumbbell in shape.
R– electron present in p orbital can have any one of the three values of
magnetic quantum number, 0,–1,+1.
17. A– Fe3+ is more stable than Fe2+ ion.
R– Fe3+ more number of unpaired electrons than Fe2+ ion.
18. A–The configuration of B cannot be 1s2,2s3.
R–Hund's rule demands that the configuration should display maximum
multiplicity.
19. A–fully filled or half–filled electronic configuration are stable.
R–they have more symmetry and more exchange energy.
20. A–The valence electronic configuration of Cr and Cu, therefore,
are3d54s 1and 3d104s 1 respectively.
R–there is extra stability associated with these electronic configurations

CS–5

The term quantum number is used to identify the variousenergy and


levels that are available to an electron.There are four types of quantum
numbers which are–Principal quantum number, azimuthal quantum
number,magnetic quantum number, spin quantum number.
21. A–An orbit cannot have more than two electrons.
R–No electrons in an atom can have same set of all four Quantum
numbers.
22.A– The energy of an electron is largely determined by its principal
quantum number.
R–The principal quantum number is a measure of the most possible
distance of finding the electron around the nucleus.
23. A–Each shell consists of one or more sub–shells.
R–The number of sub–shells in a principal shell is not equal to the value
of n.
24. A–An electron spins around its own axis, much in a similar way as
earth spins around its own axis while revolving around the sun.
R–An orbital cannot hold more than two electrons and these two
electrons should have opposite spins.
25. A– Azimuthal quantum number defines the three–dimensional shape
of the orbital.
R–For a given value of n, l can have n values ranging from 0 to n – 1.

Assertion–Reason Questions– (1x10 =10)

In the following questions (Q.1 to Q.10) a statement of Assertion


followed by a statement of Reason is given. Choose the correct answer
out of the following choice.
(A) Assertion and Reason both are correct statements but Reason is
correct explanation for Assertion.
(B)Assertion and Reason both are correct statements but Reason is not
correct explanation for Assertion.
(C)Assertion is correct statements but Reason is wrong statement.
(D)Assertion is wrong statement but Reason is correct.
1. A–The position of electron can be determined with the help of an
electron microscope.
R–The product of uncertainty in momentum and the uncertainty in the
position of an electron cannot be less than a finite limit.
2. A–The pairing of electrons in the orbitals of a particular subshell does
not occur until all the orbitals of the subshell are singly occupied.
R–Singly occupied orbitals must have the electrons with parallel spins.
3. A–Hydrogen has only one electron in its 1S orbital but it produces
several spectral lines.
R–There are many excited energy levels available in hydrogen atoms.
4. A– An orbital cannot have more than two electrons.
R–The two electrons in an orbital create opposite magnetic field.
5.A–It is impossible to determine the exact position and momentum of
an electron simultaneously.
R–The path of an electron in an atom is clearly defined.
6. A–The last electron in potassium atom enters into 4s –orbital and not
the 3d–orbital.
R– (n + l) rule is followed by for determining the orbital of the lowest
energy state.
7. A–Number of orbitals in 3rd shell is 9.
R– Number of orbitals for a particular value of n = n2.
8.A–Due to the presence of electrons in the inner shells, the electron in
the outer shell will not experience the full positive charge of the nucleus
(Ze).
R–The effect will be lowered due to the partial screening of positive
charge on the nucleus by the inner shell electrons.
9.A–The lower the value of (n + l) for an orbital, the lower is its energy.
R– Different subshells of a particular shell have different energies in case
of multi–electrons atoms.
10. A– An orbital of small size means there is more chance of finding the
electron near the nucleus.
R–Shape and orientation means the direction in which probability of
finding electron is maximum.
MCQ – (1x30=30)
1. The energy of second Bohr orbit of the hydrogen atom is –328 kJ mol–
1; hence the energy of fourth Bohr orbit would be:
(A) –41 kJ mol–1
(B) –82 kJ mol–1
(C) –164 kJ mol–1
(D) –1312 kJ mol–1
2.In which of the following Bohr‟s stationary state, the electron will be at
maximum distance from the nucleus ?
(A) IInd
(B) Ist
(C) Vth
(D) IIIrd
3.For a given principal level n = 4, the energy of its subshells is in the
order
(A) s < p < d < f
(B) s > p > d > f
(C) s < p < f < d
(D) f < p < d < s
4. Which of the following statements do not form a part of Bohr‟s model
of hydrogen atom ?
(A) Energy of the electrons in the orbits are quantized
(B) The electron in the orbit nearest the nucleus has the lowest energy
(C) Electrons revolve in different orbits around the nucleus
(D) The position and velocity of the electrons in the orbit cannot be
determined simultaneously.
5. The magnetic quantum number specifies
(A) Size of orbitals
(B) Shape of orbitals
(C) Orientation of orbitals
(D) Nuclear Stability
6. The electronic configuration of Cr (atomic number 24) is
(A) 1s2 2s22p6 3s23p6 4s23d4
(B) 1s2 2s22p6 3s23p63d5 4s1
(C) 1s2 2s22p6 3s23p6 4s24p4
(D) 1s2 2s22p6 3s23p6 4s24p6
7. The total number of electrons that can be accommodated in all
orbitals having principal quantum number 2 and azimuthal quantum
number 0 is
(A) 2 (C) 6
(B) 4 (D) 8
8. The electrons of the same orbitals can be distinguished by
(A) Principal quantum number
(B) Azimuthal quantum number
(C) Spin quantum number
(D) Magnetic quantum number
9. A body of mass 10 mg is moving with a velocity of 100 ms–1. The
wavelength of de–Broglie wave associated with it would be ( h = 6.63 ×
10–34 Js)
(A) 6.63 × 10–37 m
(B) 6.63 × 10–31 m
(C) 6.63 × 10–34 m
(D) 6.63 × 10–35 m
10. The orbitals are called degenerate when
(A) they have the same wave functions
(B) they have the same wave functions but different energies
(C) they have different wave functions but same energy
(D) they have the same energy
11. Which of the following sets of quantum numbers represents the
highest energy of an atom?
(A) n= 3, l = 0, m = 0, s = + 1/2
(B) n = 3, l = 1, m = 1, s = + 1/2
(C) n = 3, l = 2, m = 1, s = + 1/2
(D) n = 4, l = 0, m = 0, s = + 1/2
12. The number of spherical nodes in 3p orbitals are
(A) one
(B) three
(C) two
(D) None of these
13. A sub–shell with n = 5 , l = 2 can accommodate a maximum of
(A) 12 electrons
(B) 36 electrons
(C) 10 electrons
(D) 72 electrons
14. The electronic configuration of Cu2+ ion is
(A) [Ar] 4s1 3d8
(B) [Ar] 4s² 3d10 4p1
(C) [Ar] 4s1 3d10
(D) [Ar] 3d9
15. The total number of orbitals in a shell having principal quantum n is
(A) 2n
(B) n²
(C) 2n²
(D) (n + 1)
16. The line spectrum of hydrogen obtained in the visible region of light
corresponds to
(A) Lyman series
(B) Balmer series
(C) Paschen series
(D) Brackett series
17. 4d, 5p, 5f and 6p orbitals are arranged in the order of decreasing
energy. The correct option is
(A) 5f > 6p > 4d > 5p
(B) 5f > 6p > 5p > 4d
(C) 6p > 5f > 5p > 4d
(D) 6p > 5f > 4d > 5p
18. Which of the following pairs of d–orbitals will have electron density
along the axes?
(A) dz2, dxz
(B) dxz, dyz
(C) dz2, dx2 – y2
(D) dxy, dx2 – y2
19. In a given atom no two electrons can have the same values for all
the four quantum numbers. This is called
(A) Hund‟s Rule
(B) Aufbau principle
(C) Uncertainty principle
(D) Pauli‟s Exclusion principle.
20. The de–broglie equation suggest that an electron has–
(A) Particle character
(B) Wave character
(C) Particle as well as wave
(D) None of the above.
21.Which of the following orbital does not exist?
(A) 3d
(B) 3f
(C) 5p
(D) 7s.
22. Atom which does not have any neutron is–
(A) Deuterium
(B) Tritium
(C) Helium
(D) Hydrogen.
23.How many Quantum numbers are required to define the electron in
atom?
(A) 2
(B) 3
(C) 4
(D) 5.
24. Atomic number is equal to the
(A) Number of neutrons in the nucleus
(B) Number of protons in the nucleus
(C) Sum of protons and neutrons
(D) Atomic mass of the element.
25.The maximum number of electrons in s,p and d subshells are–
(A) 2 in each
(B) 2,4 and 6
(C) 2,6 and 10
(D) 2,6 and 12.
26. The electronic configuration of an atom ion can be defined by the
following
(A) Aufbau principle
(B) Pauli's exclusion principle
(C) Hund's rule
(D) All the above.
27. The correct set of quantum numbers for the unpaired electron of
chlorine atom is
(A) n=2, l=1, m=0
(B) n=2, l=1, m=1
(C) n=3, l=1, m=1
(D) n=3, l=2, m=0
28. Which of the following statement is correct
(A) The different energy levels are equally spaced
(B) The energy of the electron decreases as the value of n increases
(C) Energy of the electron in an atom is negative
(D) Bohr's theory explains the spectrum of multi electron atoms.
29. When 3d orbitals are completely filled the new electrons will enter in
(A) 4s orbitals
(B) 4p
(C) 4d
(D) None of the above.
30. An electron is moving in Bohr's orbit. Its de Broglie wavelength is λ.
What is the circumference of the forth orbit?
(A) 2/λ
(B) 2λ
(C) 4λ
(D) 3/λ
***********

ANSWER KEY–
CASE STUDY BASED

1. B MCQs–
2. D 1. B
3. D 2. C
4. B 3. A
5. C 4. D
6. D 5. C
7. A 6. B
8. A 7. A
9. B 8. C
10.A 9. C
11.D 10.D
12.D 11.C
13.A 12.A
14.A 13.C
15.A 14.D
16.B 15.B
17.B 16.B
18.B 17.B
19.A 18.C
20.A 19.D
21.A 20.C
22.A 21.B
23.C 22.D
24.B 23.B
25.B 24.B
25.C
A–R QUESTIONS 26.D
1. D
27.C
2. B
28.C
3. A
4. B 29.B
5. C 30.C
6. A
7. A
8. A
9. B
10. B
CHAPTER: 3 CLASSIFICATION OF ELEMENTS AND PERIODICITY
IN PROPERTIES
SECTION–A
Read the following passage and answer the questions given below:

1. Modern periodic table arranges the elements in the increasing


order of atomic number. It has 18 groups and 7 periods. Atomic
numbers are consecutive in a period and increases in the group in a
pattern. Elements are divided into four blocks, s–block, p–block, d–
block, and f–block based on their electronic configuration. 78% of
elements are metals, about 20 elements are non–metals, and few
elements like B, Si, Ge, As are metalloids. Metallic character increases
down the group but decreases along the period from left to right. The
physical and chemical properties vary periodically with their atomic
numbers. Periodic trends are observed in atomic size, ionization
enthalpies, electron gain enthalpies, electronegativity, and valence.
Oxides of metals are basic, some are amphoteric. Non–metals form
acidic oxides, some form neutral oxides. s–block elements are soft,
highly reactive, do not show variable oxidation states. p–block
elements are metals, non–metals as well as metalloids, show variable
oxidation states, exist as solids, liquids, and gases. d–block elements
are metals, form–coloured ions, show variable oxidation states, have
high melting and boiling points. Lanthanoids and actinoids are f–block
elements, form coloured ions. All actinoids are radioactive.

1. Which of the elements belong to the d–block but are not


transition metals
(A) Zn, Cd, Hg
(B) Sc, V, Ti
(C) Cu, Ag, Au
(D) Mn, Fe, Co
2. What are representative elements?
(A) s–block and p–block elements are called representative elements.
(B) d–block elements.
(C) f–block elements.
(D) None of these
3. What is the difference between oxidation states of p–block and d–
Block elements?
(A) In p–block elements, oxidation states differ by 2 and the lower
oxidation state is more stable whereas in d–block, oxidation state differ
by 1 and mostly higher oxidation state is more stable.
(B) In p–block elements, oxidation states differ by 1 and the lower
oxidation state is more stable whereas in d–block, oxidation state differ
by 2 and mostly higher oxidation state is more stable.
(C) In p–block elements, oxidation states differ by 2 and the higher
oxidation state is more stable whereas in d–block, oxidation state differ
by 1 and mostly lower oxidation state is more stable.
(D) None of these
4. Which group elements are most electropositive and why?
(A) Group 1 due to the largest atomic size and lowest ionization enthalpies in
respective periods.
(B) Group 1 due to the smallest atomic size and highest ionization enthalpies
in respective periods.
(C) Group 2 due to the largest atomic size and lowest ionization enthalpies in
respective periods.
(D) Group 2 due to the smallest atomic size and highest ionization enthalpies
in respective periods.
5. What happens to reactivity down the group 17?
(A) Reactivity goes on decreasing down the group as the tendency to gain
electrons decreases.
(B) Reactivity goes on increasing down the group as the tendency to gain
electrons increases.
(C) Reactivity goes on increasing down the group as the tendency to gain
electrons decreases.
(D) Reactivity goes on decreasing down the group as the tendency to
gain electrons increases.
2. The dipositive Oxidation state is the predominant valence of group 2
elements. The alkaline earth metals form compounds that are predominantly
ionic but less than the corresponding compounds of alkali metal oxides and
other compounds of their berylium and magnesium are more covalent than
those formed by the heavier and large size members. The nitrates are made by
the dissolution of the carbonates in the dilute nitric acid. Carbonates of alkaline
earth metals are insoluble in water and can be precipitated by addition of
sodium or Ammonium Carbonate solution to a solution of soluble salt of these
metals. Except for beryllium halide all other halides of alkaline earth metals are
ionic in nature. Beryllium halides essentially covalent and soluble in organic
solvents. Beryllium chloride has a chain structure.
6. Beryllium hydroxide is ––––––––––––––––––– in nature.
(A) Amphoteric
(B) Basic
(C) Acidic
(D) None of these
7. Why Beryllium does not compare well with other members of the group?
(A) Small size
(B) Ionic size
(C) Both (A) & (B)
(D) None of these
8. The basic strength of which hydroxide is maximum?
(A) LiOH
(B) NaOH
(C) Ca(OH)2
(D) KOH
9. Which of the following crystallizes as the anhydrous salt?
(A) Barium nitrate
(B) Magnesium nitrate
(C) Aluminium nitrate
(D) None of these
10. Why BeSO4 and MgSO4 are readily soluble in water?
(A) Due to greater hydration enthalpies of Be2+ and Mg2+
(B) Due to greater solvation enthalpies of Be2+ and Mg2
(C) Due to less hydration enthalpies of Be2+ and Mg2
(D) None of these

3. Classification of elements into groups and development of Periodic Law and


Periodic Table are the consequences of systematizing the knowledge gained by
a number of scientists through their observations and experiments. The German
chemist, Johann Dobereiner in early 1800‟s was the first to consider the idea of
trends among properties of elements. By 1829 he noted a similarity among the
physical and chemical properties of several groups of three elements (Triads).
The English chemist, John Alexander Newlands in 1865 profounded the Law of
Octaves. He arranged the elements in increasing order of their atomic weights
and noted that every eighth element had properties similar to the first element.
While Dobereiner initiated the study of periodic relationship, it was Mendeleev
who was responsible for publishing the Periodic Law for the first time.

11. Who first of all tried to classify elements?


(A) John Alexander Newlands
(B) Johann Dobereiner
(C) Moseley
(D) Mendeleev

12. Group of elements which makes Dobereiner‟s triads are


(A) Calcium, Strontium, Barium
(B) Lithium, Sodium, Potassium
(C) Both (A) & (B)
(D) None of these

13. Up to which element Newlands law of octavesseems to be true


(A) Potassium
(B) Calcium
(C) Barium
(D) magnesium

14.Eka–Aluminiumand Eka–Silicon named by Mendeleev were actually


(A) Molybdenum and Tellurium
(B) gallium and Germenium
(C) Barium and Silicon
(D) None of these

15. Mendeleev‟s periodic law states that


(A) The properties of the elements are aperiodic function of their
atomicweights.
(B) The properties of the elements are aperiodic function of their atomic
number.
(C) Both are correct.
(D) Both are wrong.
4. The presence of a positive charge on the nucleus is due to the protons in the
nucleus. The number of electrons in an atom is equal to the number of protons
(atomic number Z). The positive charge of the nucleus is due to protons, the
mass number of the nucleus, due to the protons and neutrons. The composition
of any atom is represented by using the normal element symbol (X) with
superscript on the left hand side as the atomic mass number (A) and subscript
Z on the left hand side as the atomic number. Isotopes are the chemical
properties of atoms that are controlled by the number of electrons, which are
determined by the number of protons in the nucleus. Isobars are atoms with
the same mass number but a different atomic number. Atoms with identical
atomic number but a different atomic mass number are known as isotopes.
Number of neutrons present in the nucleus have very little effect on the
chemical properties of an element.

In these questions, a statement of Assertion followed by statement of Reason


is given. Choose the correct answer out of the following choices:

(A) Assertion is correct, Reason is correct; Reason is a correct explanation


for Assertion.
(B) Assertion is correct, Reason is correct; Reason is not a correct
explanation for Assertion
(C) Assertion is correct, Reason is incorrect
(D) Assertion is incorrect, Reason is correct.

16.Assertion: Chlorine atoms contains 17 protons and 28 neutrons.


Reason: The total number of nucleons is termed as mass number of the atom
(A).

17.Assertion:The charge on the proton is equal but opposite to that of the


electron.
Reason:The number of protons present in the nucleus is equal to the atomic
number.

18. Assertion: Isotopes of Carbon atoms generally have 6, 7 and 8 neutrons


besides 6 protons.
Reason: Number of electrons in the hydrogen atom and sodium atom are 2
and 13 respectively.

19.Assertion: All the isotopes of a given element show same chemical


behavior.
Reason: Protons and neutrons present in the nucleus are collectively known as
nucleons.

20. Assertion: Protium, deuterium, tritium are isotopes of hydrogen.


Reason: They have same atomic number and different mass number.
SECTION–B
REASONING–ASSERTION TYPE QUESTIONS:

(A) Assertion is correct, Reason is correct; Reason is a correct explanation


for Assertion.
(B) Assertion is correct, Reason is correct; Reason is not a correct
explanation for Assertion
(C) Assertion is correct, Reason is incorrect
(D) Assertion is incorrect, Reason is correct.

21. Assertion: Generally, ionisation enthalpy increases from left to right in a


period.
Reason: When successive electrons are added to the orbitals in the same
principal quantum level, the shielding effect of inner core of electrons does not
increase very much to compensate for the increased attraction of the electron
to the nucleus.
22. Assertion: Boron has a smaller first ionisation enthalpy than beryllium.
Reason: The penetration of a 2s electron to the nucleus is more than the 2p
electron hence 2p electron is more shielded by the inner core of electrons than
the 2s electrons.

23. Assertion: Electron gain enthalpy becomes less negative as we go down a


group.
Reason: Size of the atom increases on going down the group and the added
electron would be farther from the nucleus.
24. Assertion: Smaller the size of an atom greater is the electronegativity.
Reason: Electronegativity refers to the tendency of atom to share electrons
with other atom.
25. Assertion: Atomic number of the element ununbium is 112.
Reason: Name for digits 1 and 2 is un– and bi–respectively in latin words.
26. Assertion: Hydrogen can be placed in group 1.
Reason: Hydrogen can gain an electron to achieve a noble gas arrangement.
27. Assertion: Second ionization enthalpy will be higher than the first
ionization enthalpy.
Reason: Ionization enthalpy is a quantitative measure of the tendency of an
element to lose electron.
28. Assertion: Atomic size increases along a period.
Reason: Effective nuclear charge increases as the atomic number increases
resulting in the increased attraction of electrons to the nucleus.
29. Assertion: Alkali metals have least value of ionization energy within a
period.
Reason: They precede alkaline earth metals in periodic table.
30. Assertion: Electron gain enthalpy can be exothermic or endothermic.
Reason: Electron gain enthalpy provides a measure of the ease with which an
atom adds an electron to form anion.
SECTION–C

MULTIPLE TYPE QUESTIONS:


31. The chemistry of lithium is very similar to that of magnesium even
though they are placed in different groups. Its Reason is:
(A) Both are found together in nature
(B) Both have nearly the same size
(C) Both have similar electronic configuration
(D) The ratio of their charge and size (i.e. charge density) is nearly the same
32. The element with atomic number 35 belongs to
(A) d – Block
(B) f – Block
(C) p – Block
(D) s – Block
33. The correct order of first ionization potential among following elements,
Be, B, C, N and O is
(A) B < Be < C < O < N
(B) B < Be < C < N < O
(C) Be < B < C < N < O
(D) Be < B < C < O < N
34. Representative elements are those which belong to
(A) p and d – Block
(B) s and d – Block
(C) s and p – Block
(D) s and f – Block
35. Which of the following properties generally decreases along a period?
(A) Ionization Energy
(B) Metallic Character
(C) Electron Affinity
(D) Valency
36. On the Pauling‟s electronegativity scale the element next to F is
(A) N (c)O
(B) Cl (D) Ne
37. The group number, number of valence electrons, and valency of an
element with the atomic number 15, respectively, are:
(A) 16, 5 and 2
(B) 15, 5 and 3
(C) 16, 6 and 3
(D) 15, 6 and 2
38. Which of the following oxides is amphoteric in character?
(A) SnO2 (c) SiO2
(B) CO2 (D)CaO
39. In the modern periodic table, the period indicates the value of:
(A) Atomic Number
(B) Atomic Mass
(C) Principal Quantum Number
(D) Azimuthal Quantum Number
40. Arrange S, O and Se in ascending order of electron affinity
(A) Se < S < O
(B) Se < O < S
(C) S < O < Se
(D) S < Se < O
41. Periodic classification of elements is used to examine the
(A) periodic trends in physical properties of elements
(B) periodic trends in chemical properties of elements
(C) Both (A) and (B)
(D) None of the above
42. Why the size of an anion is larger than the parent atom?
(A) Due to increased repulsion among the electrons.
(B) Due to decrease in effective nuclear charge.
(C) Due to increased in effective nuclear charge.
(D) Both (A) and (B)
43. Which of the following sequence correctly represents the decreasing
acidic nature of oxides ?
(A)Li2O >BeO> B2O3> CO2> N2O3
(B)N2O3> CO2 > B2O3>BeO> Li2O
(C)CO2> N2O3> B2O3>BeO> Li2O
(D)B2O3> CO2> N2O3> Li2O >BeO
44. Which of the following properties generally increases along a period?
(A) Ionization Energy
(B) Metallic Character
(C)Atomic radius
(D)Both B & C
45. Which of the following oxides is only basic in character?
(A) SnO2
(B) CO2
(C) SiO2
(D) CaO
46. which of the following elements are found in pitch blende?
(A) Actinium and protoactinium
(B) Neptunium and plutonium
(C) Actinium only
(D) Both (A) and (B)
47. Correct order of radii is
(A) N < Be < B
(B) F– < O2– < N3–
(C) Na < Li < K
(D) Fe3+ < Fe2+ < Fe4+
48. Set containing isoelectronic species is
(A) C2 , NO+, CN–, O2+2
−2

(B) CO, NO, O2 CN–


(C) CO2, NO2, O2, N2O
(D) CO, CO2, NO, NO2
49. Pick out the isoelectronic structures from the following
(I) CH3+ (II) H3O+
(III) NH3 (IV) CH3–
(A) I and II
(B) III and IV
(C) I and III
(D) II, III and IV
50. Eka–aluminium and Eka–silicon are known as
(A) Gallium and germanium
(B) Aluminium and silicon
(C) Iron and sulphur
(D) Neutron and magnesium
51. The general outer electronic configuration of transition elements is
(A) ns²nd1–10
(B) ns²np1(n – 1)d1–10
(C) ns²np6(n – 1)d1–10
(D) ns0–2 (n – 1)d1–10
52. Which one of the following is the Correct order of size?
(D) I > I– > I+
(B) I > I+ > I–
(C) I+ > I– > 1
(D) I– > I > I+
53. According to the periodic law of elements the variation in properties of
elements is related to their
(A)Nuclear neutron–proton number ratio
(B)Atomic masses
(C)Nuclear masses
(D)Atomic numbers
54. The correct order of electronegativity is
(A) Cl> F > O > Br
(B) F > O >Cl> Br
(C) F >Cl> Br > O
(D) O > F >Cl>Br
55. For the same value of n, the penetration power of orbital follows the
order
(A) s = p = d = f
(B) p> s > d > f
(C) f< d < p < s
(D) s< p < d < f
56. Element „X‟ forms a chloride with the formula XCl2, which is a solid with
high melting point. X would most likely be in the same group of the periodic
table as:
(A) Si (C)AI
(B) Mg (D) Na
57. Which one of the following statements is not correct about the trends in
the properties of the elements of a group on going down in a group?
(A) The chemical reactivity of metals increases.
(B) The metallic character of elements increases.
(C) The size of the atom increases.
(D) The valence electrons increase.
58. Which of the following are the characteristics of isotopes of an element?
(i) Isotopes of an element have same atomic masses
(ii) Isotopes of an element have same atomic number
(iii) Isotopes of an element show same physical properties
(iv) Isotopes of an element show same chemical properties
(A) (i), (iii) and (iv)
(B) (ii), (iii) and (iv)
(C) (ii) and (iii)
(D) (ii) and (iv)
59. Which of the following statements is not a correct statement about the
trends when going from left to right across the periods of the periodic table?
(а) The elements become less metallic in nature
(B) The number of valance electrons increases
(C) The atoms lose their electrons more easily
(D) The oxides become more acidic
60. The longest period of the Modern Periodic Table is period.
(A) first (C)fourth
(B) second (D)sixth

ANSWER KEY

Q ANS Q ANS Q ANS Q ANS Q ANS Q ANS


1 A 11 B 21 A 31 D 41 C 51 D
2 A 12 C 22 A 32 C 42 D 52 D
3 A 13 B 23 A 33 A 43 B 53 D
4 A 14 B 24 A 34 C 44 A 54 B
5 A 15 A 25 A 35 B 45 D 55 C
6 A 16 D 26 B 36 C 46 D 56 B
7 C 17 B 27 B 37 B 47 B 57 D
8 B 18 C 28 D 38 A 48 A 58 D
9 A 19 B 29 B 39 C 49 D 59 C
10 A 20 A 30 A 40 A 50 A 60 D
CHAPTER: 4 CHEMICAL BONDING AND MOLECULAR STRUCTURE

Read the following passage and answer the questions given below:

The attractive force which holds various constituents (atoms, ions, etc.)
together in different chemical species is called a chemical bond. Electrons
present in the outermost shell are known as valence electrons. Valence
electrons of an atom are represented by dots around the symbol of the
element. Valence electrons helps to calculate the common or group valence of
the element. The group valence of the elements is generally either equal to the
number of dots in Lewis symbols or 8 minus the number of dots or valence
electrons. Theory of chemical combination between atoms is known as
electronic theory of chemical bonding. It was developed by Kössel and Lewis in
1916. According to this, atoms can combine either by transfer of valence
electrons from one atom to another (gaining or losing) or by sharing of valence
electrons in order to have an octet in their valence shells. This is known as octet
rule. Lewis postulated that atoms achieve the stable octet when they are linked
by chemical bonds. Langmuir introduced the term covalent bond. Accordingto
Lewis–Langmuir theory a covalent bond is formed by sharing a pair of electron
between two bonded atoms. If covalent bond is formed by sharing a pair of
electron it is called a single covalent bond and it is represented by a line
between the two atoms
1. Atoms obtain octet configuration when linked with other atoms. This is said by
_____
(a) Lewis
(b) Kossel
(c) Langmuir
(d) Sidgwick
2. Find out the correct Lewis symbol for the atom carbon among the following
options.
(a) .C:
(b) :C.
(c) :C:
(d) .C.
3. What‟s the group valance of atoms in the halogen family?
(a) 2
(b) 1
(c) 7
(d) 8
4. Sharing or transfer of electrons from one atom to the other to attain stable
octet configuration follows _______
(a) Duet rule
(b) Octet rule
(c) Triplet rule
(d) Septet rule
5. Which of the following molecule doesn‟t involve covalent bond?
(a) O2
(b) CCl4
(c) H2O
(d) KCl

Read the following passage and answer the questions given below:

In the periodic table, the highly electronegative halogens and the highly
electropositive alkali metals are separated by the noble gases. The negative and
positive ions thus formed attain stable noble gas electronic configurations. The
noble gases (with the exception of helium which has a duplet of electrons) have
a particularly stable outer shell configuration of eight (octet) electrons,
ns2np6.The negative and positive ions are stabilized by electrostatic
attraction.Ionic bonds will be formed more easily between elements with
comparatively low ionization enthalpies and elements with comparatively high
negative value of electron gain enthalpy. Lattice enthalpy plays a key role in the
formation of ionic compounds. The Lattice Enthalpy of an ionic solid is defined
as the energy required to completely separate one mole of a solid ionic
compound into gaseous constituent ions. For example, the lattice enthalpy of
NaCl is 788 kJ mol–1. This means that 788 kJ of energy is required to separate
one mole of solid NaCl into one mole of Na+(g) and one mole of Cl– (g) to an
infinite distance. NaCl(s)→ Na+(g) + Cl– (g)
The solid crystal being three dimensional; it is not possible to calculate lattice
enthalpy directly from the interaction of forces of attraction and repulsion only.
It is determined indirectly using Born–Haber Cycle.

6.Which type of compounds show high melting and boiling points


(a)Electrovalent compounds
(b) Covalent compounds
(c) Coordinate compounds
(d) All three types of compounds have equal melting and boiling points

7.Lattice energy of an ionic compound depends upon


(a) Charge on the ion only
(b) Size of the ion only
(c) Packing of ions only
(d) Charge on the ion and size of the ion

8.Many ionic crystals dissolve in water because


(a) Water is an amphiprotic solvent
(b) Water decreasesthe interionic attraction in the crystal lattice due to
salvation
(c) The process is accompanied by a positive heat of solution
(d) Water is a high boiling liquid
9.In the transition of Al atoms to Al+++ ions there is a decrease in the
(a) Number of valence electrons
(b) Atomic weight
(c) Atomic number
(d) Equivalent weight

10.Favorable conditions for electro valency are


(a) High charge on ions, large cation, small anion
(b) High charge on ions, small cation, large anion
(c) Low charge on ions, large cation, small anion
(d) Low charge on ions, small cation, large anion

Read the following passage and answer the questions given below:

This theory provides a simple procedure to predict the shapes of covalent


molecules. Based on the repulsive interactions of the electron pairs in the
valence shell of the atoms. The shape of a molecule depends upon the
number of valence shell electron pairs (bonded or non–bonded) around
the central atom. Pairs of electrons in the valence shell repel one another
since their electron clouds are negatively charged. These pairs of
electrons tend to occupy such positions in space that minimize repulsion
and thus maximize distance between them. The valence shell is taken as
a sphere with the electron pairs localising on the spherical surface at
maximum distance from one another. A multiple bond is treated as if it is
a single electron pair and the two or three electron pairs of a multiple
bond are treated as a single super pair. Where two or more resonance
structures can represent a molecule, the VSEPR model is applicable to
any such structure. The repulsive interaction of electron pairs decrease in
the order:
lp – lp>lp –bp>bp – bp
lone pairs are localised on the central atom and each bonded pair is shared
between two atoms. As a result, the lone pair electrons in a molecule occupy
more space as compared to the bonding pairs of electrons. These results in
greater repulsion between lone pairs of electrons as compared to the lone pair –
bond pair and bond pair – bond pair repulsions.
6. Percentage of s–character in sp2 hybrid orbital is
(a) 25
(b) 50
(c) 33
(d) 75
7. Shape of XeF4 molecule is
(a) Linear
(b) Pyramidal
(c) Tetrahedral
(d) Square planar
8. Shape of methane molecule is
(a) Pyramidal
(b)Tetrahedral
(c) Octahedral
(d) Square planer
9. Hybridisation involves
(a) Mixing up of atomic orbitals
(b) Addition of an electron pair
(c) Removal of an electron pair
(d) Separation of orbitals
10. Number of lone pair(s) in XeOF4 is/are
(a) 0
(b) 1
(c) 2
(d) 3

Read the following passage and answer the ASSERTION–REASON


questions given below:

Valence bond theory was introduced by Heitlerand London (1927) and


developed further by Pauling and others. Based on the knowledge of atomic
orbitals, electronic configurations of elements, The overlap criteria of atomic
orbitals, The hybridization of atomic orbitals and the principles of variation and
superposition. A covalent bond is formed by the overlapping of atomic orbitals.
Before overlapping atomic orbitals undergo hybridization. A bond is formed
when attractive interactions balance the repulsive interactions and net energy is
lowered.Attractive forces tend to bring the two atoms close to each other
whereas repulsive forces tend to push them apart.The partial merging of atomic
orbitals is called overlapping of atomic orbitals which results in the pairing of
electrons. The extent of overlap decides the strength of a covalent bond. In
general, greater the overlap the stronger is the bond formed between two
atoms. Therefore, according to orbital overlap concept, the formation of a
covalent bond between two atoms results by pairing of electrons present in the
valence shell having opposite spins.
In the following questions a statement of Assertion followed by a
statement of Reason is given. Choose the correct answer out of the
following choices.
(a) Assertion and Reason both are correct statements and Reason is
correct explanation for Assertion.
(b) Assertion and Reason both are correct statements but Reason is
not correct explanation for Assertion.
(c) Assertion is correct statement but Reason is wrong statement.
(d) Assertion is wrong statement but Reason is correct statement.
11. Assertion:–Pi bond is never formed alone.
Reason:–Pi bond is formed by sideway overlapping of p–orbitals only.

12. Assertion:– Though the central atom of both NH3 and H2O molecules are
3
sp hybridised, yet H–N–H bond angle is greater than that of H–O–H.
Reason:– This is because nitrogen atom has one lone pair and oxygen
atom has two lone pairs.

13. Assertion:–Bond order can assume any value number including zero.
Reason:–Higher the bond order, shorter is bond length and greater is bond
energy.
14. Assertion:– The bond angle in NH3 is smaller than that of methane
Reason:–lp–bp repulsion is less in NH3
15. Assertion:– Water is a bent molecule
Reason:–It has sp3hybridisation

Read the following passage and answer the ASSERTION–REASON


questions given below:

Molecular Orbital Theory was developed by F. Hund and R.S. Mulliken in 1932.
The electrons in a molecule are present in the various molecular orbitals. The
atomic orbitals of comparable energies and proper symmetry combine to form
molecular orbitals. Molecular orbital is polycentric. The electron in a molecular
orbital is influenced by two or more nuclei depending upon the number of
atoms in the molecule. Molecular orbital is polycentric. When two atomic
orbitals combine, two molecular orbitals are formed. One is known as bonding
molecular orbital while the other is called antibonding molecular orbital. The
bonding molecular orbital has lower energy and hence greater stability than
the corresponding antibonding molecular orbital. The electron probability
distribution around a group of nuclei in a molecule is given by a molecular
orbital. The molecular orbitals like atomic orbitals are filled in accordance with
the Aufbau principle obeying the Pauli‟s exclusion principle and the Hund‟s
rule.

In the following questions a statement of Assertion followed by a


statement of Reason is given. Choose the correct answer out of the
following choices.
(a) Assertion and Reason both are correct statements and Reason is
correct explanation for Assertion.
(b) Assertion and Reason both are correct statements but Reason is
not correct explanation for Assertion.
(c) Assertion is correct statement but Reason is wrong statement.
(d) Assertion is wrong statement but Reason is correct statement.

16. Assertion:–B2 Molecule is paramagnetic.


Reason:–It has two unpaired electrons in each of p–orbitals.

17. Assertion:–Molecular nitrogen is less reactive than molecular oxygen.


Reason:– The bond length of N2 is shorter than that of oxygen.
18. Assertion:–H2Molecule more stable than HeH molecule.
Reason:– The anti–bonding electron in the molecule destabilises it.

19. Assertion:– Hydrogen combine to form H2 but Helium atoms do not


combine to form He2
Reason:–The bond order in He2 is zero whereas bond order in H2 is 1
20. Assertion:– C2 is diamagnetic while B2 is paramagnetic
Reason:–Unpaired p–orbitals are present in C2 but not in B2

In the following questions a statement of Assertion followed by a


statement of Reason is given. Choose the correct answer out of the
following choices.
(a) Assertion and Reason both are correct statements and Reason is
correct explanation for Assertion.
(b) Assertion and Reason both are correct statements but Reason is
not correct explanation for Assertion.
(c) Assertion is correct statement but Reason is wrong statement.
(d) Assertion is wrong statement but Reason is correct statement.
21. Assertion:–Sodium chloride formed by the action of chlorine gas on
sodium metal is a stable compound.
Reason:–: This is because sodium and chloride ions acquire octet in sodium
chloride formation.
22. Assertion:–Among the two O–H bonds in H2O molecule, the energy
required to break the first O–H bond and the other O–H bond is not same.
Reason:–This is because the electronic environment around oxygen is
different after breakage of one O–H bond.
23. Assertion:– LiCl is covalent whreasNaCl is ionic
Reason:–Greater the size of the cation greater is its polarising power.
24. Assertion:–BF3 is planar while NF3 is pyramidal.
Reason:–N atom is smaller than B.
25. Assertion:–Ionic compounds tend to be non–volatile.
Reason:–Ionic compounds are solid.
26. Assertion:–SF6 is a stable molecule.
Reason:–A stable molecule must have 8 electrons around the central atom i.e.
octet rule must be satisfied.
27. Assertion:–H–S–H bond angle in in H2S is closer to 900,but H–O–H bond
angle in H2O is 104.50.
Reason:–l.p.–lp repulsion is stronger in H2S than in H2O.
28. Assertion:– is a weaker base than .
Reason:– The lone pairs of electrons on N atom in is in the sp2–orbital
while in ., it is an sp3–orbital.
29. Assertion:–Both o–hydroxybenzaldehyde and p– hydroxybenzaldehyde
have same molecular weight and show H–bonding.
Reason:–Melting point of p– hydroxybenzaldehyde is more.
30. Assertion:–H2O is the only hydrate of group–16 which is liquid at
ordinary temperature.
Reason:–In ice ,each oxygen atom is surrounded by two covalent bonds and
two H–bonds.
MULTIPLE CHOICE TYPEOF QUESTIONS:
31. The types of hybrid orbitals of nitrogen in , and respectively
are expected to be
(a) sp, sp3 and sp2
(b) sp, sp2 and sp3
(c) sp2, sp and sp3
(d) sp2, sp3 and sp
32. Which of the following species has tetrahedral geometry?
(a)
(b)
(c)
(d) H3O+
33. Which molecule/ion out of the following does not contain unpaired electrons?
(a)
(b) O2
(c)
(d) B2
34. Which of the following angle corresponds to sp2 hybridisation?
(a) 90°
(b) 120°
(c) 180°
(d) 109°
35. Which of the following statement is not correct from the view point of
molecular orbital theory?
(A) Be2 is not a stable molecule.
(B) He2 is not stable but is expected to exist.
(C) Bond strength of N2 is maximum amongst the homonuclear diatomic molecules
belonging to the second period.
(D) The order of energies of molecular orbitals in N2 molecule is
σ 2s<σ*2s < σ2pz<(π2px = π2py ) < (π*2px = π*2py ) <σ*2pz
36. Which of the following options represents the correct bond order :
(a) > O2 >
(b) < O2 <
(c) > O2 <
(d) < O2 >
37. Which of the following have identical bond order?
(i) (ii) (iii) (iv)
(a) (i) and (iii)
(b) (i) and (ii)
(c) (i) and (iv)
(d) (ii) and (iii)
38. Which of the following species have the same shape?
(i) CO2 (ii) CCl4 (iii) O3 (iv)
(a) (i) and (iii)
(b) (iii) and (iv)
(c) (ii) and (iii)
(d) (i) and (iv)
39. Diamagnetic species are those which contain no unpaired electrons. Which
among the following are diamagnetic?
(i) N2 (ii) (iii) O2 (iv)
(a) (i) and (iv)
(b) (i) and (iii)
(c) (ii) and (iii)
(d) (iii) and (iv)
40. Which of the following statements are not correct?
(i) NaCl being an ionic compound is a good conductor of electricity in the solid
state.
(ii) In canonical structures there is a difference in arrangement of atoms.
(iii) Hybrid orbitals form stronger bonds than pure orbitals.
(iv) VESPER Theory can explain the square planar geometry of XeF4
(a) (iii) and (iv)
(b) (i) and (ii)
(c) (ii) and (iii)
(d) (i) and (iv)
41. Shape of PCl5 is
(a) Octahedral
(b) Square pyramidal
(c) Trigonal Planar
(d) Trigonalbipyramidal
42. Which of the following statements is correct?
(a) In the formation of dioxygen from oxygen atoms 10 molecular orbitals will
be formed.
(b) All the molecular orbitals in the dioxygen will be completely filled.
(c) Total number of bonding molecular orbitals will not be same as total
number of anti bonding orbitals in dioxygen.
(d) Number of filled bonding orbitals will be same as number of filled anti
bonding orbitals.
43. The force which holds atoms together in an electrovalent bond is
(a) Vander Waal's force
(b) Dipole attraction force
(c) Electrostatic force of attraction
(d) All the above
44. The main reaction during electrovalent bond formation is
(a) Redox reaction
(b) Substitution reaction
(c) Addition reaction
(d) Elimination reaction
45. Which type of bonding exists in Li2O and CaF2 respectively
(a) Ionic, ionic
(b) Ionic, covalent
(c) Covalent, ionic
(d) Coordinate, ionic
46. An atom with atomic number 20 is most likely to combine chemically with
the atom whose atomic number is
(a) 11
(b) 14
(c) 16
(d) 10
47. The nature of bonding in graphite is
(a) Covalent
(b) Ionic
(c) Metallic
(d) Coordinate
48. The covalency of nitrogen in HNO3 is
(a) 0
(b) 3
(c) 4
(d) 5
49. Hydrogen chloride molecule contains a
(a) Covalent bond
(b) Double bond
(c) Coordinate bond
(d) Electrovalent bond
50. Which molecule has the largest dipole moment
(a) HCl
(b) HI
(c) HBr
(d) HF
51. Which of the following will show least dipole character
(a) Water
(b) Ethanol
(c) Ethane
(d) Ether
52. Which of the following molecules will show dipole moment
(a) Methane
(b) Carbon tetrachloride
(c) Chloroform
(d) Carbon dioxide
53. If the electron pair forming a bond between two atoms A and B is not in the
centre, then the bond is
(a) Single bond
(b) Polar bond
(c) Non–polar bond
(d) pi bond
54. Polarization is the distortion of the shape of an anion by an adjacently
placed cation. Which of the following statements is correct
(a) Maximum polarization is brought about by a cation of high charge
(b) Minimum polarization is brought about by a cation of low radius
(c) A large cation is likely to bring about a large degree of polarization
(d) A small anion is likely to undergo a large degree of polarization
55. The bonds between P atoms and Cl atoms in PCl5 are likely to be
(a) Ionic with no covalent character
(b) Covalent with some ionic character
(c) Covalent with no ionic character
(d) Ionic with some metallic character
56. Which of the following is not correct
(a) A sigma bond is weaker than pi bond
(b) A sigma bond is stronger than pi bond
(c) A double bond is stronger than a single bond
(d) A double bond is shorter than a single bond
57. Strongest bond formed, when atomic orbitals
(a) Minimum overlap
(b) Maximum overlap
(c) Overlapping not done
(d) None of them
58. In N2 molecule, the atoms are bonded by
(a) One sigma, Two pi
(b) One sigma, One pi
(c) Two sigma, One pi
(d) Three pi bonds
59. Shape of methane molecule is
(a) Tetrahedral
(b) Pyramidal
(c) Octahedral
(d) Square planer
60. The hydrogen bond is strongest in
(a) Water
(b) Ammonia
(c) Acetic acid
(d) Hydrogen fluoride
ANSWER KEY

Q ANS Q ANS Q ANS Q ANS Q ANS Q ANS Q ANS


1 a 11 c 21 a 31 c 41 b 51 c 61 a
2 c 12 d 22 a 32 b 42 b 52 a 62 b
3 b 13 b 23 b 33 a 43 b 53 d 63 a
4 b 14 a 24 a 34 b 44 a 54 a 64 a
5 d 15 b 25 c 35 b 45 b 55 d 65 d
6 a 16 c 26 a 36 b 46 d 56 c
7 d 17 a 27 a 37 a 47 a 57 c
8 b 18 a 28 c 38 c 48 c 58 b
9 a 19 c 29 b 39 b 49 a 59 a
10 c 20 b 30 b 40 d 50 a 60 b
CHAPTER: 8 REDOX REACTION
Read the following passage and answer the questions given below:

Q1. In order to keep track of electron shifts in the chemical reactions involving
formation of covalent compounds, and more practical method of using
oxidation number has been developed. In this method it is always assumed
that there is a complete transfer of electron from a less electronegative atom
to a more electronegative atom. It may be emphasized that the assumption of
electron transfer leads to the simple description of redox reactions.
1. The oxidation state of Cl2, O3, P4, S8, Na, Mg, Al is:
a) +2
b) +3
c) 0
d) –1

2. In the reaction, when H2 combines with O2 to form water, there is an


electron transfer from O2 to H2O
a) H to O
b) O to H
c) No electron transfer
d) All the above

3. Non metallic elements have


a) Positive oxidation number
b) Negative oxidation number
c) Positive and negative oxidation number
d) No defined oxidation number

4. Which of the following arrangement represent increasing oxidation number


of central atom?
a) CrO2–, ClO3–, CrO42–, MnO4–
b) ClO3– ,CrO42– , MnO4– , CrO2–
c) CrO2– , ClO3– , MnO4– , CrO42–
d) CrO42–, MnO4– , CrO2– , ClO3–

5. In which of the following compounds, an element exhibits two different


oxidation states.
a) NH2OH
b) NH4NO3
c) N2H4
d) N 3H

II. Oxidation reaction involves loss of electrons; reduction reaction involves


gain of electrons. The reaction in which a species disproportionate in two
oxidation states (lower and higher) is called disproportionation reaction
6. Which of the following statement is wrong?
a. An acidified K2Cr2O7 paper on being exposed to SO2 turns green
b. Mercuric chloride and stannous chloride cannot exist as such.
c. Iron turning on addition to CuSO4 solution decolourises
d. [CuI4]2– is formed but [CuCl4]2– is not.
7. Which of the following statement is wrong?
a. Acidified KMnO4 solution decolourises on the addition of sodium oxalate
b. In the reaction between Br2 and CsI, Br2 is an oxidising agent and CsI is a
reducing agent
c. In the reaction of 2K2S2O3 + I2→2KI + K2S4O6, the change in the oxidation
number of S is 0.5
d. C has the same oxidation number in both CH4 and CO2
8. which of the following statement is correct?
a. An element in the lowest oxidation state acts only as a reducing agent.
b. An element in the highest oxidation state acts only as a reducing agent.
c. The oxidation state of V in Rb4K(HV10O28) is +4
d. The oxidation number and valency of Hg in calomel is +1.

9. Which of the following statement is wrong?


a. The algebraic sum of the oxidation numbers of all the atoms in an ion is
zero
b. The oxidation number is an arbitrary number. It can have positive,
negative, zero or fractional values.
c. When negative ion changes to neutral species, the process is oxidation
d. The oxidation number of phosphorous can vary from –3 to +5.

10. Which of the following is not a disproportionation reaction?


a. OH– + Br2 → Br– + BrO3–
b. Cu2O + 2H+→ Cu + Cu2+ + H2O
c. CN–→ CO32– + NO3–
d. (CN)2 + 2OH–→ CN– + CNO– + H2O

III. Certain material such as turpentine oil, unsaturated organic compounds,


phosphorous, metals such as Zn and Pb, etc., can absorb oxygen from air in the
presence of H2O, which is converted to H2O2. This is called auto oxidation.
Intermolecular redox reactions are those in which one molecule is oxidised and
the other is reduced. Intramolecular redox reactions are those in which one
atom of a molecule is oxidised and the other atom is reduced.
11.Which of the following statement about the reaction is/ are correct?
2AuCl4– + 3Zn → 2Au +3Zn2+ + 8Cl–
a. 2AuCl4– is reduced to Au
b. Zn is oxidised to Au
c. Cl– is a spectator ion.
d. It is an intermolecular redox reaction.
12.which of the following reaction is/are disproportionation reaction(s)?
a. Cl2 + 2OH–→Cl– +ClO– + H2O
b. HCHO + OH–→ CH3OH + HCOO–
c. MgCO3→MgO + CO2
d. 2HCuCl2→ Cu +Cu2+ + 4Cl– +2H+
13.which of the following reaction is/are intramolecular redox reaction(s)?
a. 2Mn2O7→ 4MnO2 + 3O2
b. K3[Fe(CN)6] + 30H2O → Fe3+ + 6CO2 + 6NO3– +60H+ +60e–
c. 2HgO → 2Hg + O2
d. PhCHO→ PhCH2OH + PhCOONa
14.which of the following reaction is/are intermolecular redox reaction(s)?
a. 5KI + KIO3 + 6HCl → 3I2 + 6KCl + 3H2O
b. Fe + N2H4→ NH3 + Fe(OH)2
c. NO3– +H2S +H2O + H+→ NH4+ +HSO4–
d. Cr2O72– +2OH–→ 2CrO42– +H2O

15. Which of the following reactions has/have spectator ions?


a. Zn + CuSO4→ ZnSO4 + Cu
b. KIO3 + KI + H2SO4→ KI3 + K2SO4 + H2O
c. 2KMnO4 + 10KCl + 8H2SO4→ 5Cl2 + 2MnSO4 + 8H2O + K2SO4
d. [CrCl6]3– + Zn → [ZnCl4]2– + Cr2+

Part– B CASE BASESD ASSERTION AND REASON

The following questions given below consist of an "Assertion" (A)


and "Reason" (R)Type questions. Use the following Key to choose
the appropriate answer.
(A)If both (A) and (R) are true, and (R) is the correct
explanation of (A).
(B) If both (A) and (R) are true but (R) is not the correct
explanation of (A).
(C)If (A) is true but (R) is false.
(D)If (A) is false but (R) is true.
(E) IF both (A) and (R) are false

IV. Originally, the term oxidation was used to describe the addition of oxygen
to an element or a compound. Because of the presence of dioxygen in the
atmosphere (~20%), many elements combine with it and this is the principal
Reason why they commonly occur on the earth in the form of their oxides. The
following reactions represent oxidation processes according to the limited
definition of oxidation:
2 Mg(s) + O2(g)→ 2 MgO(s)(1)
S (s) + O2(g) → SO2(g) (2)
In reactions (1) and (2), the elements magnesium and sulphur are oxidised on
account of addition of oxygen to them. Similarly, methane is oxidised owing to
the addition of oxygen to it. CH4(g) + 2O2(g) →CO2(g) + 2H2O (l) (3). A careful
examination of reaction (3) in which hydrogen has been replaced by oxygen
prompted chemists to reinterpret oxidation in terms of removal of hydrogen
from it and, therefore, the scope of term oxidation was broadened to include
the removal of hydrogen from a substance. The following illustration is another
reaction where removal of hydrogen can also be cited as an oxidation reaction.
16. Assertion (A):H2O2 acts only as an oxidising agent.
H2O2→ H2O + O
Reason (R): All peroxide behaves as oxidising agent only
17. Assertion (A): KMnO4 is a stronger oxidising agent than K2Cr2O7.
Reason (R): this is due to increasing stability of lower species which they are
reduced.
18. Assertion (A): SO2 and Cl2 are both bleaching agents.
Reason (R): both are reducing agent.

19. Assertion (A): F2 undergoes deprotonation reaction


Reason (R): Fluorine shows both positive and negative oxidation state.

20. Assertion (A): Snreact with HCl to produce H2 gas.


Reason (R): Sn is better reducing agent than hydrogen.

V. Oxidation number denotes the oxidation state of an element in a compound


ascertained according to a set of rules formulated on the basis that electron pair
in a covalent bond belongs entirely to more electronegative element. It is not
always possible to remember or make out easily in a compound/ion, which
element is more electronegative than the other. Therefore, a set of rules has
been formulated to determine the oxidation number of an element in a
compound/ion. If two or more than two atoms of an element are present in the
molecule/ion such as Na2S2O3/Cr2O72–, the oxidation number of the atom of that
element will then be the average of the oxidation number of all the atoms of
that element.
21. Assertion (A): O2 is the stronger reducing agent than F2
Reason (R): F2 is more electronegative.

22. Assertion (A): O3 can act as an oxidising agent as well as reducing


agent, but SO2 can act only as an oxidising agent.
Reason (R): The oxidation number of O in O3 is zero, and the oxidation
number of S in SO2 is +4

23. Assertion (A): The two atoms in Fe3O4 have different oxidation number.
Reason (R): Fe2+ ion decolourises KMnO4 solution.

24. Assertion (A): HNO3 acts an oxidising agent while HNO2 acts both as an
oxidising agent and reducing agent.
Reason (R): The oxidation number of N is maximum in HNO3

25. Assertion (A): A reaction between Fe and I2 occur but a reaction


between Fe2+ and I– does not occur.
Reason (R): Fe is better reducing agent than I–

Part– C ASSERTION AND REASON


The following 10 questions given below consist of an "Assertion" (A) and
"Reason" (R)Type questions. Use the following Key to choose the appropriate
answer.
(A)If both (A) and (R) are true, and (R) is the correct explanation of
(A).
(B) If both (A) and (R) are true but (R) is not the correct explanation of
(A).
(C) If (A) is true but (R) is false.
(D) If (A) is false but (R) is true.
(E) IF both (A) and (R) are false
26. Assertion (A): In Basic medium colour of K2Cr2O7 is changed from orange
to yellow.
Reason (R): In basic medium K2Cr2O7 is changed in chromate ion.

27.Assertion (A): HClO4 is only oxidizing agent.


Reason (R): Cl is most electronegative element in H, Cl& O.

28. Assertion (A): The oxidation number of S in Na2S2O3 is +6


Reason(R): The Na2S2O3 can undergo disproportionation reaction in the
acidified solution.

29. Assertion (A): For every oxidation–reduction reaction oxidant and


reductant must be different molecules or ions.
Reason (R): Oxidation and reduction always occur simultaneously.

30.Assertion(A): SO2and Cl2 both are bleaching agent.


Reason(R): SO2 bleaches by reduction whereas Cl2 bleaches by oxidation

31. Assertion(A): Redox titrations, involving KMnO4 in acid medium, cannot


be done in presence of hydrochloric acid.
Reason(R): Hydrochloric acid does not provide enough H+ ions for the reaction

32. Assertion(A): N atom has two different oxidation states in NH4NO2.


Reason (R): One N atom has –ve oxidation number as it is attached with less
electronegative H atom and other has +ve oxidation number as it is
attached with more electronegative atom

33.Assertion(A): SO2 can be used as reductant as well as oxidant.


Reason (R): The oxidation number of S is +4 in SO2 which lies in between its
minimum (–2) and maximum (+6)values.

34.Assertion(A): Equivalent weight of KMnO4 in acidic medium isM/5.


Reason (R): In acidic medium KMnO4→ MnO2.

35. Assertion: Redox titrations, involving KMnO4 in acid medium, cannot be


done in presence of hydrochloric acid.
Reason (R): Hydrochloric acid does not provide enough H+ ions for the
reaction.

Part–D MULTIPLE CHOICE QUESTIONS

36. The oxidation number of Cl in Cl2O7 is


(a) + 7
(b) + 5
(c) + 3
(d) – 7
37. Which of the following processes does not involve either oxidation or
reduction?
(a) Formation of slaked lime from quick lime
(b) Heating Mercuric Oxide
(c) Formation of Manganese Chloride from Manganese oxide
(d) Formation of Zinc from Zinc blende
38. One mole of N2H4 loses ten moles of electrons to form a new compound A.
Assuming that all the nitrogen appears in the new compound, what is the
oxidation state of nitrogen in A? (There is no change in the oxidation state of
hydrogen.)
(a) –1
(b) –3
(c) +3
(d) +5
39. The oxidation state of Cr in Cr (CO)6 is
(a) 0
(b) 2
(c) 2
(d) 6
40. Which of the following processes does not involve oxidation of iron?
(a) Formation of Fe(CO)5 from Fe.
(b) Liberation of H2 from steam by iron at high temperature.
(c) Rusting of iron sheets.
(d) Decolourisation of blue CuSO4 solution by iron.
41. The number of moles of KMnO4 reduced by one mole of KI in alkaline
medium is
(a) One
(b) Two
(c) Five
(d) One fifth
42. The oxidation number of Xe in BaXeO6 is
(a) 8
(b) 6
(c) 4
(d)10
43. The oxidation number of Mn is maximum in
(a) MnO2
(b) K2MnO4
(c) Mn3O4
(d) KMnO4
44. KMnO4 reacts with oxalic acid according to the equation 2MnO4– + 5C2O42–
+ 16H+ → 2Mn2+ +10CO2 + 8H2O Here 20 mL of 0.1 M KMnO4 is equivalent to.
(a) 50 mL of 0.5 M C2H2O4
(b) 20 mL of 0.1 M C2H2O4
(c) 20 mL of 0.5 M C2H2O4
(d) 50 mL of 0.1 M C2H2O4
45.Which of the following reactions does not involve either oxidation or
reduction?
(a) VO2+ → V2O3 (b) Na → Na+
2– –2
(c) CrO 4 → Cr2O7 (d) Zn2+ →Zn
46. Which of the compounds can exist together?
(a) HgCl2, SnCl2
(b) FeCl3, KI
(c) FeCl3, SnCl2
(d) FeCl2, SnCl2
47.H2O2 changes Cr2O72– ion to CrO5 in an acidic medium, the oxidation
state of Cr in CrO5 is
(a) +6
(b) +5
(c) –10
(d) +3
48. Which among the following is the strongest oxidising agent?
(a) H2O2
(b) O3
(c) K2Cr2O7
(d) KMnO4
49. Which among the following compounds is the most reducing
compound?
(a) H2S
(b) HNO2
(c) SnCl2
(d) H2SO3
50.In which of the following complex, the oxidation number of Fe is
+1?
(a) Fe4[Fe(CN)6]3
(b) [Fe(H2O)5NO]SO4
(c) [FeBr4]–
(d) [Fe(H2O)6]2–
51. Find the oxidation state of I in H4IO6–
(a) +7
(b) +5
(c) +1
(d) –1
52. During a reaction of oxalic acid, potassium chlorate and sulphuric
acid, the oxidation number of which of the element undergoes a
maximum change
(a) H
(b) S
(c) C
(d) Cl

53. H2SO4 acts as a strong oxidising agent. In which of the reaction, is


it not acting as an oxidising agent?
(a) C + 2H2SO4 → CO2 + 2SO2 + 2H2O
(b) CaF2 + 2H2SO4 → CaSO4 + 2HF
(c) S + 2H2SO4 → 3SO2 + H2O
(d) Cu + 2H2SO4 → CuSO4 + SO2 + 2H2O

54. One mole of ferrous oxalate requires____ moles of MnO4– to get


oxidised completely in an acidic medium
(a) 0.6 moles
(b) 0.4 moles
(c) 0.2 moles
(d) 7.5 moles

55. Zn gives H2 gas with H2SO4 and HCl but not with HNO3 because

(a) Zn acts as an oxidising agent when it reacts with HNO3


(b) HNO3 is a weaker acid than H2SO4 and HCl
(c) In electrochemical series, Zn is above hydrogen
(d) NO3– is reduced in preference to hydronium ion

56. In the reaction


3Br2 + 6CO32– + 3H2O → 5Br– + BrO3– + 6HCO3–

(a) bromine is oxidised and carbonate is reduced


(b) bromine is reduced and water is oxidised
(c) bromine is neither reduced nor oxidised
(d) bromine is both reduced and oxidised

57. Which of the following cannot function as an oxidising agent?

(a) I–
(b) Si(s)
(c) NO3–(aq)
(d) Cr2O72–

58. The oxidation number of Pt in [Pt(C2H4)Cl3]– is

(a) +1
(b) +2
(c) +3
(d) +4
59. The value of x & y in the following redox reaction–
xCl2 + 6OH– → ClO3− + yCl– + 3H2O are
(a) x = 2, y = 4
(b) x = 5, y = 3
(c) x = 3, y = 5
(d) x = 4, y = 2
60. Which of the following is not a reducing agent?
(a) SO2
(b) H2O2
(c) CO2
(d) NO2–
61. The oxidation number of iron in Fe3O4 is
(a) +2
(b) +3
(c) 8/3
(d) 2/3
62. Oxidation state of osmium (Os) in OsO4 is
(a) +7
(b) + 6
(c) +4
(d) + 8
63. The oxidation number of sulphur in S8, S2F2, H2Srespectively are
(a) 0, +1, –2
(b) +2,+1,–2
(c) 0, +1, +2
(d) –2, +1,–2
64. Among the following, identify the species with an atom in + 6
oxidation state:
(a) MnO4–
(b) Cr(CN)63−
(c) NiF62−
(d) CrO2Cl2
65. The reaction 3ClO– (aq) → CIO3– (aq) + 2Cl–(aq) is an example of–
(a) Oxidation reaction
(b) Reduction reaction
(c) Disproportionation reaction
(d) Decomposition reaction

ANSWER KEY
Q ANS Q ANS Q ANS Q ANS Q ANS Q ANS Q ANS
1 C 11 all 21 D 31 C 41 B 51 A 61 C
2 B 12 a,b,c 22 D 32 A 42 D 52 D 62 D
3 C 13 a,c 23 B 33 A 43 D 53 B 63 A
4 A 14 a,b,c 24 A 34 C 44 A 54 B 64 A
5 B 15 all 25 A 35 C 45 C 55 D 65 C
6 D 16 E 26 A 36 A 46 D 56 D
7 D 17 A 27 C 37 A 47 A 57 A
8 A 18 C 28 D 38 C 48 B 58 A
9 A 19 E 29 D 39 A 49 A 59 C
10 C 20 A 30 A 40 A 50 B 60 C
CHAPTER 9 HYDROGEN
CASE BASED MCQ QUESTIONS AND ANSWERS–1
Hydrogen is enormously attractive fuel because it is environmentally clean.
“Hydrogen economy” is a new field in which it is thought that our energy needs
can met by gaseous, liquid and solid hydrogen. Since hydrogen is not a
naturally occurring substance like coal, oil or natural gas, energy must
expended to produce hydrogen before it can be used. Current researches
therefore on finding cheaper methods for extracting hydrogen.
1.Which fuel does produce least environmental pollution?
(a)Kerosene oil
(b) Hydrogen
(c)Wood
d)Coal
2.If an isotope of hydrogen has two neutrons in its atom,its atomic number and
mass number will be respectively be:
(a)2 and 1
(b) 3 and 1
(c)1 and 1
(d) 1 and
3.Which of the following gas is lightest ?
(a) Oxygen
(b)Ammonia
(c)Hydrogen
(d)Helium
4.Which isotope of hydrogen is radioactive in nature ?
(a) Protium
(b) Deuterium
(c) Deuterium and Tritium
(d) Tritium only
5.Liquid hydrogen has been used as a rocket fuel because of :
(a)High thrust
(b)Its reaction with oxygen is highly exothermic
(c)Small space it occupies
(d) All these are correct

CASE BASED MCQ QUESTIONS AND ANSWERS–2


Water is most common and abundant compound of hydrogen. Water
occupies the major part of the earth.it is essential of life. water has the
ability to dissolve a number of substances. Certain salts of calcium and
magnesium make water hard.Hard water forms scum with soap.Hard
water is unsuitable for laundry,it is unsuitable for boilers as well.
Therefore, removal of these salts from water is necessary.This is called
softening of water.

6.The reagent commonly used to determine hardness of water titrimetrically is


(a) Oxalic acid
(b) Disodium salt of EDTA
(c) Sodium citrate
(d) Sodium thiosulphate
7.Polyphosphates are used as water softening agents because they
(a) Form soluble complexes with an ionic species
(b) Precipitate an ionic species
(c) Form soluble complexes with cationic species
(d) Precipitate cationic species.
8.Hardness of water may be temporary or permanent. Permanent hardness is
due to the presence of
(i) Chlorides of Ca and Mg in water
ii) Sulphates of Ca and Mg in water
(iii) Hydrogen carbonates of Ca and Mg in water
(iv) Carbonates of alkali metals in water
(a) (i)and (ii)
(b) (ii)and (iii)
(c) (iii)and (iv)
(d)(i)and (iv)
9.Permutit is:
(a) Hydrated sodium aluminium silicate
(b) Sodium hexa phosphate
(c)Sodium silicate
(d) Sodium meta aluminate
10.In a calgon process of softening of water ,which of the following is used .
(a) Sodium hexameta–phosphate
(b)Cation exchange resins
(c) Anion exchange resins
(d)Hydrated sodium aluminium silicate.

CASE BASED MCQ QUESTIONS AND ANSWERS–3


Dihydrogen combines with a number of elements to form binary compounds
called hydrides ,their general formula being EHx where E represents the
element and x the number of hydrogen .Depending upon the physical and
chemical properties ,the hydrides have been divided in to three broad
categories.Besides three main categories of hydrides ,certain hydrides are
better known as polymeric hydrides and complex hydrides.

11. Which one is ionic hydride in nature?


(a) CrH
(b) NH3
(c) H2O
(d) NaH
12. Which group forms hydride?
(a) 7
(b) 6
(c) 8
(d) 9
13. Elements of which of the following group(s) of periodic table do not form
hydrides.
(a) Groups 7, 8, 9
(b) Group 13
(c) Groups 15, 16, 17
(d) Group 14
14.Which of the following hydride is electron deficient hydride.
(a) CH4
(b) NH3
(c) H2O
(d) B2H6
15. Metal hydrides are ionic, covalent or molecular in nature. Among LiH, NaH,
KH, RbH, CsH, the correct order of increasing ionic character is
(i) LiH>NaH>CsH> KH>RbH
(ii) LiH<NaH< KH <RbH<CsH
(iii) RbH>CsH>NaH> KH >LiH
(iv) NaH>CsH>RbH>LiH> KH

CASE BASED ASSERTIONREASON QUESTIONS AND ANSWERS–4


Q.1Water is colourless and tasteless liquid. The unusual properties of water in
the condensed phase (liquid and solid)are due to the presence of extensive
hydrogen bonding between water molecules. This leads to high freezing point,
high boiling point, high heat of vaporization and high heat of fusion in
comparison to H2S and H2Se .In gaseous state ,water exists as discrete
molecule. When water freezes, it forms ice which is the crystalline form of water
Depending upon the conditions employed for freezing of water, nine crystalline
forms of ice are known.
In the following questions a statement of Assertion (A) followed by a
statement of Reason (R) is given.
Answer (a) If Both A and R are true and R is the correct explanation of A.
(b) If Both A and R are true but R is not the correct explanation of A.
(c) If A is true but R is false.
(d) If Both A and R are false.
Choose the correct option out of the choices given above.
16.Assertion (A) : Density of ice is less than water.
Reason (R) : Ice has open cage structure
17.Assertion :H2O is only hydride of 16 group which is a liquid at ordinary
temperature.
Reason :In ice ,each oxygen atom is surrounded by two covalent bonds and
two hydrogen bonds .
18.Assertion : The critical temperature of water is higher than that of O2 .
Reason: The H2O molecule has dipole moment .
19.Assertion :Water is a universal solvent .
Reason: Water possesses a high dielectric constant .
20. Assertion : At its melting point, ice is lighter than water because
Reason : H2O molecules are more closely packed in solid state .

CASE BASED ASSERTIONREASON QUESTIONS AND ANSWERS–5


Hydrogen is the first element in the periodic table. However ,a proper
position could not be assigned to it either in Mendeleev „s periodic table or
Long form of periodic table. In some properties, it resembles alkali metals
.In some properties ,hydrogen resembles halogens. In some yet another
properties ,hydrogen differ from both alkali and halogens. Thererfore it is
not justified to include hydrogen either with the alkali metals of group 1 or
halogen group of 17. That is why hydrogen is sometimes referred to as a
“rogue element “ .
21.Assertion:Hydrogen like alkali metals exhibit electropositive character .
Reason:Hydrogen can lose electron to form H+ ion .
22.Assertion:The compounds of hydrogen with halogen are covalent in nature.
Reason:Hydrogen shows an oxidation state of –1.
23.Assertion:Hydrogen resembles halogen of 17 group.
Reason: Ionisation enthalpy of hydrogen is much higher than alkali metals..
24.Assertion : H+ does not exist freely and is always associated with other
atoms or molecules.
Reason : Loss of the electron from hydrogen atom results in nucleus (H+ ) of
~ 1.5 × 10–3 pm size. This is extremely small as compared to normal atomic
and ionic sizes of 50 to 200 pm.
25.Assertion : Hydrogen combines with other elements by losing, gaining or
sharing of electrons.
Reason : Hydrogen forms electrovalent and covalent bonds with other
elements .

ASSERTION AND REASON QUESTIONS

In the following questions a statement of Assertion (A) followed by a


statement of Reason (R) is given.
Answer (a) If Both A and R are true and R is the correct explanation of A.
(b) If Both A and R are true but R is not the correct explanation of A.
(c) If A is true but R is false.
(e) If Both A and R are false.

26.Assertion : Temporary hardness can be removed by boiling.


Reason :On boiling the soluble bicarbonates change to
carbonates which being insoluble, get precipitated.
27.Assertion : Water can act as acid as well as base.
Reason : Water can accept as well as donate H+ ion.
28.Assertion : Some metals like platinum and palladium, can be used as
storage media for hydrogen.
Reason : Platinum and palladium can absorb large volumes of hydrogen.
29.Assertion : Demineralized water does not contain any ion .
Reason :Permutit process for water softening gives demineralized water .
30.Assertion :Chlorine reacts more rapidly with H2 than withD2.
Reason :D–Cl bond is stronger than H–Cl bond.
31. Assertion :Dihydrogen oxidizes sodium to sodium hydride.
Reason : Hydrogen can act only as reducing agent .
32.Assertion : Nascent hydrogen can discharge the pink colour of KMnO4
solution . Reason : Nascent is much more reactive than dihydrogen .
33.Assertion : Calgon is used for removing Ca2+ and Mg2+ ions .
Reason :Calgon forms precipitate with Ca2+ and Mg2+ ions
34.Assertion : Berylium hydride is a covalent hydride .
Reason :The electronegativity difference Between Be and H is very high .
35.Assertion : The process of adsorption of hydrogen on palladium is known
as occlusion .
Reason :The adsorbed hydrogen is more active than ordinary hydrogen .
MULTIPLE CHOICE QUESTIONS
36.Hydrogen will not reduce
(a) heated cupric oxide
(d) heated ferric oxide
(c) heated stannic oxide
(d) heatedaluminium oxide
37. Which of the following terms is not correct for hydrogen ?
(a) Its molecule is diatomic
(b) It exists both as H+ and H– in different chemical compounds
(c) It is the only species which has no neutrons in the nucleus
(d) Heavy water is unstable because hydrogen is substituted by its isotope
deuterium
38.The sum of the number of neutrons and protons in all the
three isotopes of hydrogen is
(a) 6
(b) 5
(c) 4
(d) 3
39.Which of the following is not true?
(a) D2O freezes at lower temperature than H2O
(b) Reaction between H2 and Cl2 is much faster than D2 and Cl2
(c) Ordinary water gets electrolysed more rapidly than D2O
(d) Bond dissociation energy of D2 is greater than H2
40.Heavy water reacts respectively with CO2, SO3, P2O5 and
N2O5 to give the compounds :
(a) D2CO3, D2SO4, D3PO2, DNO2
(b) D2CO3, D2SO4, D3PO4, DNO2
(c) D2CO3, D2SO3, D3PO4, DNO2
(d) D2CO3, D2SO4, D3PO4, DNO3
41.Which of the following is electron–deficient?
(a) (BH3)2
(b) PH3
(c) (CH3)2
(d) (SiH3)2.6
42.The method used to remove temporary hardness of
water is
(a) synthetic resins method
(b) Calgon‟smethod
(c) Clark‟s method
(d) ion–exchange method.
43.The number of hydrogen bonded water molecule(s)
associated with CuSO4. 5H2O is
(a) 3
(b) 1
(c) 2
(d) 5
44.The freezing point of heavy water is
(a) 0°C (c) 4°C
(b) 3.8°C (d) 1°C
45.The critical temperature of water is higher than that of O2 because the H2O
molecule has
(a) Fewer electrons than oxygen
(b) Two covalent bonds
(c) v–shape
(d) Dipole moment
46. Which of the following compounds is used for water softening?
(a) Ca3(P04)2
(b) Na3P04
(c) Na6P6018
(d) Na2HP04
47. Which of the following statement(s) is/are correct in the case of heavy
water?
(a) Heavy water is used as a moderator in nuclear reactor.
(b) Heavy water is more effective as solvent than ordinary water.
(c) Heavy water is more associated than ordinary water.
(d) Heavy water has lower boiling point than ordinary water.
48. Which of the following statements are correct?
(a) Hydrides of group 13 act as Lewis acids.
(b) Hydrides of group 14 are electron deficient hydrides.
(c) Hydrides of group 14 act as Lewis acids.
(d) Hydrides of group 15 act as Lewis bases.
49.Which of the following is correct about heavy water ?
(a) Water at 4°C having maximum density is known as
heavy water
(b) It is heavier than water (H2O)
(c) It is formed by the combination of heavier isotope of
hydrogen with oxygen
(d) None of these
50.D2O is preferred to H2O, as a moderator, in nuclear reactors
because
(a) D2O slows down fast neutrons better
(b) D2O has high specific heat
(c) D2O is cheaper
(d) None of these
51. Elements of which of the following group do not form
hydrides?
(a) Alkali metals
(b) Halogens
(c) Alkaline earth metals
(d) Noble gases
52.Which of the following metal evolves hydrogen on reacting
with cold dilute HNO3 ?
(a) Mg
(b) Al
(c) Fe
(d) Cu
53.Which of the following is formed when zinc reacts with
sodium hydroxide?
(a) Hydrogen gas
(b) Sodium zincate
(c) Zinc oxide
54. LiAlH4 is used as :
(a) An oxidizing agent
(b) A reducing agent
(c) A mordant
(d) A water softener
55.Permanent hardness of water is due to the presence of
(a) bicarbonates of sodium and potassium
(b) chlorides and sulphates of sodium and potassium
(c) chlorides and sulphates of calcium and magnesium
(d) bicarbonates of calcium and magnesium
56.The H–O–H angle in water molecule is about
(a) 90º
(b) 180º
(c) 102.5°
(d) 104.5º
57.Why does H+ ion always get associated with other atoms
or molecules?
(a) Ionisation enthalpy of hydrogen resembles that of
alkali metals.
(b) Its reactivity is similar to halogens.
(c) It resembles both alkali metals and halogens.
(d) Loss of an electron from hydrogen atom results in a
nucleus of very small size as compared to other atoms
or ions. Due to small size it cannot exist free.
58.Which of the following statements is correct ?
(a) Hydrogen has same IP as alkali metals
(b) Hydrogen has same electronegativity as halogens
(c) It has oxidation number of –1 and +1
(d) It will not be liberated at anode
59.Which one of the following statement about water is false?
(a)Water is oxidized to oxygen during photosynthesis .
(b)Water can act both an acid and base ,
(c)There is extensive intra molecular hydrogen bonding in the condensed phase
(d)Ice formed by heavy water sinks in normal water.
60.Which of the following disproportionates when treated with water ?
(a)SO3
(b)F2
(c)N2
(d)Cl2
61.Which of the following fuel is used for runnning the automobiles first time in
the history of India during October
2005?
(a) D2O
(b) H2O2
(c) D2
(d) H2
62.Whentwo ice cubes are pressed over each other, they unite to form one
cube. Which of the following forces is responsible to hold them together ?
(a) Hydrogen bond formation
(b) Vander Waals forces
(c) Covalent attraction
(d) Ionic interaction
63.Which of the following sequence of T and F is correct for given statements?
Here T stands for true and F stands for false statement
(i) The H–H bond dissociation enthalpy is highest for a single bond between two
atoms of any element
(ii) H2 is relatively inert at room temperature.
(iii) Hydrogen combines with almost all the elements due to its incomplete
orbital
(iv) The atomic hydrogen is produced at high temperature in an electric arc or
under UV radiations.
(a) TTTT
(b) FTFT
(c) FTTT
(d) FTTF
64.When zeolite (hydrated sodium aluminium silicate) is treated with hard
water the sodium ions are exchanged with
(a) H+ ions
(b) Ca2+ ions
(c) SO42– ions
(d) OH– ions

65.What is formed when calcium carbide reacts with heavy water?


(a) C2D2
(b) CaD2
(c) Ca2D2O
(d) CD2

ANSWER KEY

Q ANS Q ANS Q ANS Q ANS Q ANS Q ANS Q ANS


1 B 11 D 21 A 31 C 41 A 51 D 61 D
2 D 12 B 22 B 32 A 42 C 52 A 62 A
3 C 13 A 23 B 33 D 43 B 53 D 63 A
4 D 14 D 24 A 34 C 44 B 54 B 64 B
5 D 15 B 25 A 35 B 45 D 55 C 65 A
6 B 16 A 26 A 36 D 46 C 56 D
7 C 17 B 27 C 37 D 47 A,C 57 D
8 A 18 A 28 A 38 A 48 A,D 58 C
9 A 19 A 29 C 39 A 49 C 59 C
10 A 20 C 30 B 40 D 50 D 60 D
CHAPTER–12
Organic Chemistry: Some Basic Principles and
Techniques
SECTION–A
CASE BASED MCQ QUESTIONS

Q.1 READ THE PASSESGE GIVEN BELOW AND GIVE THE APROPRIATE
ASWER OF MCQ.
Organic compounds are vital for sustaining life on earth and include
complex molecules like genetic information bearing deoxyribonucleic acid
(DNA) and proteins that constitute essential compounds of our blood,
muscles and skin. Organic compounds appear in materials like clothing,
fuels, polymers, dyes and medicines. These are some of the important
areas of application of these compounds.
Science of organic chemistry is about two hundred years old. Around the
year 1780, chemists began to distinguish between organic compounds
obtained from plants and animals and inorganic compounds prepared
from mineral sources. Berzilius, a Swedish chemistproposed that a „vital
force‟ was responsible for theformation of organic compounds. However,
this notionwas rejected in 1828 when F. Wohler synthesisedanorganic
compound, urea from an inorganic compound,ammoniumcyanate.

Ammonium cyanate Urea


The pioneering synthesis of acetic acid by Kolbe (1845) and that of
ethane by Berthelot (1856) showed conclusively that organic compounds
could be synthesised from inorganic sources in a laboratory.
Q. 1 Vital force is found in?
(A) DNA
(B) NH4OH
(C) NaCl
(D)NH4CNO

Q.2 What is the vital force in the vital force theory?


(A) A mysterious power that creates organic compounds
(B) Any organic compound
(C) Any living organism
(D) The energy needed to create organic compounds from inorganic
compounds

Q.3 From which compound, the F. Wohler synthesised an organic


compound.
(A)Ammonium hydroxide
(B)Ammonium carbonate
(C)Ammonium cyanate
(D)ammonium acetate
Q.4 Which of the following was NOT a principle of the vital force theory?
(A) Organic compounds cannot be made in the laboratory from inorganic
compounds
(B)Synthesis of organic compounds requires a vital force
(C) Only living organisms (God–given) contain this vital force
(D)Energy could be isolated to create the vital force

Q.5 which one is an inorganic compound?


(A)Ammonium cyanate
(B)Ethane
(C)Acetic acid
(D)DNA

Q.2 READ THE PASSESE GIVEN BELOW AND GIVE THE APROPRIATE
ASWER OF MCQ.
The purpose of the IUPAC system of nomenclature is to establish an
international standard of naming compounds to facilitate communication.
The goal of the system is to give each structure a unique and
unambiguous name, and to correlate each name with a unique and
unambiguous structure.
IUPAC nomenclature is based on naming a molecule‟s longest chain of
carbons connected by single bonds, whether in a continuous chain or in a
ring. All deviations, either multiple bonds or atoms other than carbon and
hydrogen, are indicated by prefixes or suffixes according to a specific set
of priorities.Here is a simple list of rules to follow.

 Identify the longest carbon chain. This chain is called the parent
chain.
 Identify all of the substituents (groups appending from the parent
chain).
 Number the carbons of the parent chain from the end that gives
the substituents the lowest numbers. When compairing a series of
numbers, the series that is the "lowest" is the one which contains
the lowest number at the occasion of the first difference. If two or
more side chains are in equivalent positions, assign the lowest
number to the one which will come first in the name.
 If the same substituent occurs more than once, the location of each
point on which the substituent occurs is given. In addition, the
number of times the substituent group occurs is indicated by a
prefix (di, tri, tetra, etc.).
 If there are two or more different substituents they are listed in
alphabetical order using the base name (ignore the prefixes). The
only prefix which is used when putting the substituents in
alphabetical order is iso as in isopropyl or isobutyl. The prefixes
sec– and tert– are not used in determining alphabetical order
except when compared with each other.
 If chains of equal length are competing for selection as the parent
chain, then the choice goes in series to:
a) the chain which has the greatest number of side chains.
b) the chain whose substituents have the lowest– numbers.
c) the chain having the greatest number of carbon atoms in the
smaller side chain.
d)the chain having the least branched side chains.
 A cyclic (ring) hydrocarbon is designated by the prefix cyclo– which
appears directly in front of the base name.

Q.1 Which of the following is the correct IUPAC name?


(A) 3–Ethyl–4, 4–dimethylheptane
(B) 4,4–Bis(methyl)–3–ethylheptane
(C) 5–Ethyl–4, 4–dimethylheptane
(D) 4,4–Dimethyl–3–ethylheptane

Q.2 The correct decreasing order of priority for the functional groups of
organic
compounds in the IUPAC system of nomenclature is:
(A) –CHO, –COOH, –SO3H, –CONH2
(B)–SO3H, –COOH, –CONH2, –CHO
(C)–COOH, –SO3H, –CONH2, –CHO
(D)–CONH2, –CHO, –SO3H, –COOH

Q.3The IUPAC name of CH3– CH = CH – C ≡ CH is:


(A) pent–3–en–4–yne
(B) pent–3–en–1–yne
(C)pent–2–en–4–yne
(D) pent–2–en–3–yne

Q.4 IUPAC name of compound CH3CH = CH – CHO is


(A) But–2–enal.
(B) Prop–2–enal
(C) But–1–enal
(D) But–2–enol

Q.5 which one is wrong pair


(A) Amides –carbamoyl
(B) Carboxylic–acid –carboxy
(C) Nitriles –cyano
(D) Aldehydes– oxo
Q.3 READ THE PASSESGE GIVEN BELOW AND GIVE THE APROPRIATE
ASWER OF MCQ.
The knowledge of fundamental concepts of molecular structure helps in
understanding and predicting the properties of organic compounds. Also,
you already know that tetravalence of carbon and the formation of
covalent bonds by it are explained in terms of its electronic configuration
and the hybridisation of s and p orbitals. It may be recalled that
formation and the shapes of molecules like methane (CH4), ethene
(C2H4), ethyne (C2H2) are explained in terms of the use of sp3, sp2and
sphybrid orbitals by carbon atoms in the respective molecules.
Hybridisation influences the bond length and bond enthalpy (strength) in
compounds. The sphybrid orbital contains more s character and hence it
is closer to its nucleus and forms shorter and stronger bonds than thesp3
hybrid orbital. The sp2 hybrid orbital is intermediate in s character
between spand
sp3 and, hence, the length and enthalpy of the bonds it forms, are also
intermediate between them. The change in hybridisation affects the
electronegativity of carbon. The greater the s character of the hybrid
orbitals, the greater is the electronegativity. Thus, a carbon atom having
ansphybrid orbital with 50% s character is more electronegative than
that possessing sp2 or sp3hybridised orbitals. Thisrelative
electronegativity is reflected in several physical and chemical properties
of the molecules concerned. Thus, in H2C=CH2 molecule all the atoms
mustbe in the same plane. The p orbitals are mutually parallel and both
the p orbitals are perpendicular to the plane of the molecule.
Rotation of one CH2 fragment with respect to other interferes with
maximum overlap of p orbitals and, therefore, such rotation about
carbon–carbon double bond (C=C) is restricted. The electron charge
cloud of the π bond is located above and below the plane of bonding
atoms. This results in the electrons being easily available to the attacking
reagents. In general, π bonds provide the mostreactivecentres in the
molecules containing multiple bonds.

Q.1 The property of self–linkage among identical atoms to form long


chain compounds is known as:

(A) Catenation
(B) Isomerisation
(C) Superposition
(D) Halogenation

Q.2 Which organic species has only one type of hybridized carbon?
(A)CH3– C*H – CH3 * indicates unpaired electron on carbon
(B)CH3–CH2–CH3
(C) CH3– C*= CH2 * indicates unpaired electron on carbon
(D) CH3–C≡CH
Q.3 how many sigma and pi bonds are present in CH3– CH = CH – C ≡
CH
(A)sigma bond–9, pi bond–4
(B) sigma bond–9, pi bond–3
(C) sigma bond–10, pi bond–3
(D) sigmabond–10, pi bond–4

Q.4 which one is right


(A)σ= π
(B)σ< π
(C)π> σ
(D)σ> π
Q.5 carbon have
(A)tetravalence
(B)petravalence
(C)zerovalence
(D) all of these

SECTION–B
Q.1 CASE BASE ASSERTION REASONQUESTIONS
The phenomenon of existence of two or morecompounds possessing the
same molecularformula but different properties is known asisomerism.
Such compounds are called asisomers.
Compounds having the same molecularformula but different structures
(manners inwhich atoms are linked) are classified asstructural isomers.
The compounds that have the sameconstitution and sequence of covalent
bondsbut differ in relative positions of their atomsor groups in space are
called stereoisomers.This special type of isomerism is called
asstereoisomerism and can be classified asgeometricaland optical
isomerism
A group or a series of organic compounds each containing a
characteristic functional group forms a homologous series and the
members of the series are called homologues. The members of a
homologous series can be represented by general molecular formula and
the successive members differ from each other in molecular formula by a
–CH2 unit. There are a number of homologous series of organic
compounds. Some of these are alkanes, alkenes, alkynes, haloalkanes,
alkanols, alkanals, alkanones, alkanoic acids, amines etc.
The questions given below are Assertion (A) and Reason (R). Use the
following key to select the correct answer.
(A) If both Assertion and Reason are correct and Reason is correct
explanation forAssertion.
(B) If both Assertion and Reason are correct but Reason is not correct
explanationforAssertion.
(C) If Reason is correct but Assertion is incorrect.
(D) If both Assertion and Reason are incorrect.
Q.1 Assertion: But–1–ene and 2–Methylprop–1–ene are position
isomers.
Reason: Position isomers have same molecular formula but different
arrangement of carbon atoms.

Q. 2 Assertion: All the carbon atoms in But–2–ene are sp2 hybridized.


Reason: All the carbon atoms of But–2–ene lie in one plane.

Q. 3 Assertion: Alkanes having more than three carbon atoms exhibit


chain
isomerism.
Reason: All carbon atoms in alkanes are sp–hybridised.

Q.4 Assertion: In CH2= C = CH2, all the carbon atoms are


2
sp hybridised.
Reason: All the hydrogen atoms lie in one plane.
Q. 5 Assertion: Butane and 2–Methylbutane are homologues.
Reason: Butane is a straight chain alkane while 2–Methylbutane is
Branchedchain alkane.

Q.2 CASE BASED ASSERTION REASON QUESTIONS

In an organic reaction, the organic molecule(also referred as a substrate)


reacts with anappropriate attacking reagent and leads to theformation of
one or more intermediate(s)andfinally product(s)
Substrate is that reactant which supplies carbon to the new bond and the
other reactant is called reagent. If both the reactants supply carbon to
the new bond then choice is arbitrary and in that case the molecule on
which attention is focused is called substrate.In such a reaction a
covalent bondbetween two carbon atoms or a carbon and some other
atom is broken and a new bond isformed. A sequential account of each
step, describing details of electron movement, energetics during bond
cleavage and bond formation, and the rates of transformation of
reactants into products kinetics) is referred to as reaction mechanism.
The knowledge of reaction mechanism helps in understanding the
reactivity of organic compounds and in planning strategy for their
synthesis. A species having a carbon atom possessing sextext of
electrons and a positive charge is called a carbocation (earlier called
carbonium ion). The CH+3ion is known as a methyl cation or methyl
carbonium ion. arbocations are classified as primary, secondary or
tertiary depending on whether one, two or three carbons are directly
attached to the positively charged carbon. Some other examples of
carbocations are: CH3C+H2(ethyl cation, a primary carbocation),
(CH3)2C+H (isopropyl cation, a secondary carbocation), and (CH3)3C+
(tert–butyl cation, a tertiary carbocation).Carbocations are highly
unstable and reactivespecies. Alkyl groups directly attached to the
positively charged carbon stabilise the carbocations due to inductive and
hyperconjugation effects, The observed order of carbocation stability is:
C+H3< CH3C+H2< (CH3)2C+H < (CH3)3C+. These carbocations have
trigonal planar shape with positively charged carbon being sp2hybridised.
Thus, the shape of C+H3 may be considered as being derived from the
overlap of three equivalent C(sp2) hybridised orbitals with 1s orbital of
each of the three hydrogen atoms. Each bond may be represented as
C(sp2)–H(1s) sigma bond. The remainingcarbon orbital is perpendicular
to the molecular plane and contains no electrons. Alkyl radicals are
classified as primary, secondary, or tertiary. Alkyl radical stability
increases as we proceedfromprimary to tertiary:

Organic reactions, which proceedbyhomolytic fission are called free


radical orhomopolaror nonpolar reactions.

The questions given below are Assertion (A) and Reason (R). Use the
following key to select the correct answer.
(A) If both Assertion and Reason are correct and Reason is correct
explanation for Assertion.
(B) If both Assertion and Reason are correct but Reason is not correct
explanation for Assertion.
(C) If Reason is correct but Assertion is incorrect.
(D) If both Assertion and Reason are incorrect.

Q.1Assertion: Tertiary carbonations are generally formed more easily


than primary carbocations.
Reason: Hyperconjugation as well as inductive effect due to
additionalAlkylgroups stabilize tertiary carbocations.
Q.2 Assertion: Alkyl carbanions like ammonia have pyramidal shape.
Reason: The carbon atom carrying negative charge has an octet of
electrons.
Q.3 Assertion: Carbocations are planar in nature.
Reason: Carbocations are sp2Hybridised.
Q.4 Assertion : All the carbon atoms in H2C = C = CH2 are
2
sp hybridised
Reason : In this molecule all the carbon atoms are attached to each
other by double bonds.
Q.5 Assertion : Pent– 1– ene and pent– 2– ene are position isomers.
Reason : Position isomers differ in the position of functional group or a
substituent.
SECTION–C
ASSERTION REASON QUESTIONS

Q.1Assertion: IUPAC name of compound CH3CH = CH – CHO is But–2–


enal.
Reason: Functional group gets preference over multiple in IUPAC name
ofa compound.
(A) Both A and R are correct and R is the correct explanation of A.
(B) Both A and R are correct but R is not the correct explanation of A.
(C) Both A and R are not correct.
(D) A is not correct but R is correct.

Q.2 Assertion: Energy of resonance hybrid is equal to the average of


energiesof all canonical forms.
Reason: Resonance hybrid cannot be presented by a single structure.
(A) Both A and R are correct and R is the correct explanation of A.
(B) Both A and R are correct but R is not the correct explanation of A.
(C) Both A and R are not correct.
(D) A is not correct but R is correct.

Q.3 Assertion: Hex– 1– ene and Hex– 2– ene are functional group
isomers.
Reason: Functional group isomers have the same molecular formula but
different functional groups.
(A) Both A and R are correct and R is the correct explanation of A.
(B) Both A and R are correct but R is not the correct explanation of A.
(C) Both A and R are not correct.
(D) A is not correct but R is correct.

Q.4 Assertion: free radical are obtained by mean of homolytic cleavage.


Reason: In homolytic cleavage, each of the atom acquires one of
thebonding electrons.
(A) Both A and R are correct and R is the correct explanation of A.
(B) Both A and R are correct but R is not the correct explanation of A.
(C) Both A and R are not correct.
(D) A is not correct but R is correct.

Q.5 Assertion : Alkenes and cycloalkanes series of hydrocarbons have


same general formula.
Reason : Either insertion of a double bond or formation of a ring reduce
the number of hydrogen atoms of corresponding alkane by 2.
(A) Both A and R are correct and R is the correct explanation of A.
(B) Both A and R are correct but R is not the correct explanation of A.
(C) Both A and R are not correct.
(D) A is not correct but R is correct.
Q.6 Assertion: Alkyl group act as electron donors when attached to a
pi–system.
Reason: it is due to hyperconjugation.
(A) Both A and Rare correct and R is the correct explanation of A.
(B) Both A and R are correct but R is not the correct explanation of A.
(C) Both A and R are not correct.
(D) A is not correct but R is correct.

Q.7 Assertion: pi– boand stronger than sigma bond.


Reason: The extent of overlapping in sigma bond is greater than in pi–
bond.
(A) Both A and R are correct and R is the correct explanation of A.
(B) Both A and R are correct but R is not the correct explanation of A.
(C) Both A and R are not correct.
(D) A is not correct but R is correct.

Q.8Assertion : The greater the s–character of hybrid orbital, the greater


is the
electronegativity.
Reason: the change in hybridization influence the electronegativity of
carbon.
(A) Both A and R are correct and R is the correct explanation of A.
(B) Both A and R are correct but R is not the correct explanation of A.
(C) Both A and R are not correct.
(D) A is not correct but R is correct.

Q.9 Assertion: The sp hybrid orbital is closer to its nucleus.


Reason: The sp hybrid orbital has less s character.
(A) Both A and R are correct and R is the correct explanation of A.
(B) Both A and R are correct but R is not the correct explanation of A.
(C) Both A and R are not correct.
(D) A is not correct but R is correct.

Q.10 Assertion : Though the central atom of both NH3 and H2O molecule
are sp3 hybridized, yet H−N−H bond angle is greater than that of H−O−H.
Reason : This is because nitrogen atom has one lone pair and oxygen
atom has two lone pairs.

(A) Both A and R arecorrect and R is the correct explanation of A.


(B) Both A and R are correct but R is not the correct explanation of A.
(C) Both A and R are not correct.
(D) A is not correct but R is correct.
SECTION–D
MULTIPLE CHOICE QUESTION

Q.1If two compounds have the same empirical formula but different
molecular formula they must have
(A) Different percentage composition
(B) Different molecular weight
(C) Same viscosity
(D) Same vapour density
Q.2 Identify the chiral molecule among the following:
(A) Isopropyl alcohol
(B) 2–pentanol
(C) 1–bromo 3–butene
(D) Isobutyl alcohol
Q.3 0.0833mol of carbohydrate of empirical formula CH2O contain 1g of
hydrogen. The molecular formula of the carbohydrate is
(A) C5H10O5
(B) C3H4O3
(C) C12H22O11
(D) C6H12O6
Q.4 The displacement of electrons in a multiple bond in the presence of
attacking reagent is called
(A) Inductive effect
(B) Electromeric effect
(C) Resonance
(D) Hyper conjugation.
Q.5 Which of the following cannot be represented by resonance
structures?
(A) Dimethyl ether
(B) Nitrate anion
(C) Carboxylate anion
(D) Toluene
Q.6 An organic compound which produces a bluish green coloured flame
on heating in presence of copper is
(A) Chlorobenzene
(B) Benzaldehyde
(C) Aniline
(D) Benzoic acid
Q.7 Which one is strongest acid among following options?
(A) CH2FCOOH
(B) CH2ClCOOH
(C) CHCl2COOH
(D) CHF2COOH
Q.8 Inductive effect involves
(A) displacement of σ electrons
(B) delocalization of π electrons
(C) delocalization of σ–electrons
(D) displacement of π–electrons
Q.9 Which of the following behaves both as a nucleophile and as an
electrophile?
(A) CH3C ≡ N
(B) CH3OH
(C) CH2 = CHCH3
(D) CH3NH2
Q.10 I.U.P.A.C. name of 4–isopropyl–m–xylene is
(A) l–Isopropyl–2, 4–Dimethyl benzene
(B) 4–Iso propyl–m–Xylene
(C) l–Isopropyl–3, 5–Dimethyl benzene
(D) 4–Isopropyl–3, 5–Dimethyl benzene.
Q.11 The I.U.P.A.C. name of

(A) 3–Methyl cyclohexene


(B) 1–methyl cylohex–2–ene.
(C) 6–methyl cyclohexene
(D) 1–methyl cyclohex5–ene.
Q.12 The I.U.P.A.C. name of CH3COCH(CH3)2 is
(A) 3–methyl–2–butanone
(B) Isopropyl methyl ketone
(C) 2–methyl–3–butanone
(D) 4–methyl isopropyl ketone
Q.13 Which of the following compounds possess the C–H bond with the
lowest bond dissociation energy?
(A) Toluene
(B) Benzene
(C) n–Pentane
(D) 2, 2–Dimethyl propane
Q.14 Name of the compound given below is

(A) 4–ethyl–3–methyloctane
(B) 3 –methyl–4–ethyloctane
(C) 2, 3–diethylheptane
(D) 3– ethyl–2–methyl heptane.
Q.15 The C–H bond distance is longest in
(A) C2H2
(B) C2H4
(C) C2H6
(D) C2H2Br2
Q.16 Which of the following represents the given mode of hybridisation
sp²–sp²–sp–sp from left to right?
(A) CH2 = CH–C ≡ N
(B) HC ≡ C – C = CH
(C) H2C = C = C = CH2

(D)
Q.17 What is the decreasing order of stability of the ions?
(I) CH3–CH–CH3
(II) CH3–CH–OCH3
(III) CH3–CH–CO–CH3
(A) I > II > III
(B) II > III > I
(C) III > I > II
(D) II > I > III
Q.18 Which of the following is an electrophile?
(A) H2O
(B) NH3
(B) AlCl3
(D) C2H5NH2
Q.19 The most stable carbanion among the following is

(A)

(B)

(C)

(D)
Q.20 The increasing order of electron donating inductive effect of alkyl group is :
(A) −H<−CH3<−C2H5<−C3H7
(B) −H>−CH3>−C2H5>−C3H7
(C) −H<−C2H5<−CH3<−C3H7
(D) −H>−C2H5>−CH3>−C3H7
Q.21 The I.U.P.A.C. name of the following compound is

(A) 3–Ethyl–2–hexene
(B) 3–Propyl–2–bexene
(C) 3–Propyl–3–hexene
(D) 4–Ethyl–4–hexene
Q.22 The I.U.P.A.C name of the compond is

(A) 3, 3, 3– Trimethyl propene


(B) 1,1, 1–Trimethyl–2–pentane
(C) 3, 3–Dimethyl–1–butene
(D) 2, 2–Dimethyl–3–butane.
Q.23 Whichof the following is correct regarding the–I. Effect of
substituents?
(A) –NR2 < –OR < F–
(B) –NR2 > –OR < –F
(C) –NR2 < –OR < –F
(D) –NR2 > –OR > –F
Q.24The I.U.P.A.C. name of CH3 – CH = CH COO C2H5 is
(A) Ethyl but–1 –enoate
(B) Ethyl but–2–enoate
(C) Ethyl prop–2–enoate
(D) None of these.
Q.25 CH3CH2Cl undergoeshomolytic fission to produce
(A) CH3C*H2 &Cl*
(B) CH3CH+2 &Cl–
(C) CH3CH+2 &Cl*
(D) CH3CH2 &Cl–
Q.26 The I.U.P.A.C. name of CH3COCH(CH3)2 is
(A) 3–methyl–2–butanone
(B) Isopropyl methyl ketone
(C) 2–methyl–3–butanone
(D) 4–methyl isopropyl ketone
Q.27 In which of the following, functional group isomerism is not
possible?
(A) Alcohols
(B) Aldehydes
(C) Alkyl halides
(D) Cyanides
Q.28 Which of the following can act as an electrophile?
(A) CN–
(B) OH–
(C) H2O
(D) BF3
Q.29 Isomers of a compound must have :
(A) Same physical properties
(B) Same chemical properties
(C) Same structural properties
(D) Same molecular weight
Q.30 The type of isomerism not found in alkenes is :
(A) Chain isomerism
(B) Geometrical isomerism
(C) Metamerism
(D) Position isomerism

Answer key

SECTION–A
Case based question.
[Q.1]. 1.A 2.A 3.C 4.D 5. A
[Q.2] 1. A 2.C 3.B 4.A 5. D
[Q.3] 1.A 2.B 3.C 4.D 5. A

SECTION–B
Case based A/R Questions
[Q.1] 1. D 2.C 3.C 4.D 5. B

[Q.2] 1.A 2.B 3.A 4. C 5. A

SECTION–C
A/R Questions

Q.1 A Q.6 A
Q.2 D Q.7 D
Q.3 D Q.8 A
Q.4 A Q.9 C
Q.5 A Q.10 A
SECTION–D
MCQ
Q.1. (B) Different molecular weight
Q.2(D) Isobutyl alcohol
Q.3(D) C6H12O6
Q.4(B) Electromeric effect
Q.5(A) Dimethyl ether
Q.6(D) Benzoic acid
Q.7(D) CHF2COOH
Q.8(A) displacement of σ electrons
Q.9(A) CH3C ≡ N
Q.10(A) l–Isopropyl–2, 4–Dimethyl benzene
Q.11(A) 3–Methyl cyclohexene
Q.12(A) 3–methyl–2–butanone
Q.13(A) Toluene
Q.14(D) 3– ethyl–2–methyl heptane.
Q.15(C) C2H6
Q.16(A) CH2 = CH–C ≡ N
Q.17(D) II > I > III
Q.18(C) AlCl3
Q.19(D)
Q.20(A)
Q.21(A) 3–Ethyl–2–hexene
Q.22(C) 3, 3–Dimethyl–1–butene
Q.23(A) –NR2 < –OR < F–
Q.24(B) Ethyl but–2–enoate
Q.25(A) CH3C*H2 &Cl*
Q.26(A) 3–methyl–2–butanone
Q.27(C) Alkyl halides
Q.28(D) BF3
Q.29(D) Same molecular weight
Q.30 (C) Metamerism
MODEL TEST PAPER –2021–22

BLUE PRINT
S Case
NO. Name of chapter study A–R MCQ Total
1 Some basic concepts of Chemistry 1x3 1x3 6
2 Structure of Atom 1x5 1x7 12
3 Periodic classification 1x6 6
4 Chemical bonding and molecular structure 1X9 9
5 Redox reaction 1X4 4
6 Hydrogen 1X4 4
7 Organic chemistry: some basic principles and techniques 1X14 14
3 5 47 55
MODEL TEST PAPER
SUBJECT– CHEMISTRY TERM I
CLASS – XI
MM: 35 TIME: 90
MINS
General Instructions:
1. The Question Paper contains three sections.
2. Section A has 25 questions. Attempt any 20 questions.
3. Section B has 24 questions. Attempt any 20 questions.
4. Section C has 6 questions. Attempt any 5 questions.
5. All questions carry equal marks.
6. There is no negative marking.

SECTION A
This section consists of 25multiple choice questions with overall
choice to attempt any 20 questions. In case more than desirable
number of questions are attempted, ONLY first 20 will be considered
forevaluation.

1. How many orbitals can have the following set of quantum numbers, n = 3, l
= 1, m1 = 0 ?
(A) 3
(B) 1
(C) 4
(D) 2

2. Electronic configuration of the outer shell of the element Gd with atomic


number 64 is
(A) 4f4 5d5 6s1
(B) 4f3 5d5 6s2
(C) 4f5 5d4 6s1
(D) 4f7 5d1 6s2
3. Number of unpaired electrons in N2+
(A) 3
(B) 1
(C) 2
(D) 0
4. A 0.66 kg ball is moving with a speed of 100 m/s. Find its wavelength
(A) 6.6 × 10–34 m
(B) 6.6 × 10–32 m
(C) 1.0 × 10–32 m
(D) 1.0 × 10–35 m
5. Number of protons, neutrons and electrons in the element 89X231 is
(A) 89, 89, 242
(B) 89, 142, 89
(C) 89, 71, 89
(D) 89, 231, 89
6. Rutherford‟s experiment which established the nuclear model of the atom
used a beam of
(A) β–particles which impinged on a metal foil and got absorbed
(B) γ–rays, which impinged on a metal foil and got absorbed
(C) helium atoms which impinged on a metal foil and got reflected
(D) helium nuclei which impinged on a metal foil and got scattered
7. Azimuthal quantum number determines the
(A) size
(B) spin
(C) orientation
(D) angular momentum of orbitals
8. The chemistry of lithium is very similar to that of magnesium even though
they are placed in different groups. Its Reason is:
(A) Both are found together in nature
(B) Both have nearly the same size
(C) Both have similar electronic configuration
(D) The ratio of their charge and size (i.e. charge density) is nearly the same
9. The element with atomic number 35 belongs to
(A) d – Block
(B) f – Block
(C) p – Block
(D) s – Block
10. The correct order of first ionization potential among following elements, Be,
B, C, N and O is
(A) B < Be < C < O < N
(B) B < Be < C < N < O
(C) Be < B < C< N < O
(D) Be < B < C < O < N
11. In the modern periodic table, the period indicates the value of:
(A) Atomic Number
(B) Atomic Mass
(C) Principal Quantum Number
(D) Azimuthal Quantum Number
12. In the long form of the periodic table, the valence shell electronic
configuration of 5s²5p4 corresponds to the element present in:
(A) Group 16 and period 6
(B) Group 17 and period 6
(C) Group 16 and period 5
(D) Group 17 and period 5
13. Modern periodic table is based on
(A) Atomic number
(B) Atomic weight
(C) Both a and b
(D) None of these
14.The hybridization state of sulphur in SO2 molecule is :
(A) sp²
(B) sp³
(C) sp
(D) sp³d
15. Among the following mixtures, dipole–dipole as the major interaction, is
present in
(A) benzene and ethanol
(B) acetonitrile and acetone
(C) KCl and water
(D) benzene and carbon tetrachloride
16. Which of the following types of hybridisation leads to three dimensional
geometry of bonds around the carbon atom?
(A) sp
(B) sp²
(C) sp³
(D) None of these
17. An atom of an element A has three electrons in its outermost orbit and that
of B has six electrons in its outermost orbit. The formula of the compound
between these two will be
(A) A3B6
(B) A2B3
(C) A3B2
(D) A2B
18.The number of types of bonds between two carbon atoms in calcium carbide:
(A) TWO SIGMA, TWO PI
(B) ONE SIGMA, TWO PI
(C) ONE SIGMA, ONE PI
(D) TWO SIGMA, ONE PI
19.NaCl has the following bond type :
(A) Ionic
(B) Covalent
(C) Coordinate
(D) None of these
20.Shape of BeCl2 is :
(A)Linear
(B) Square planar
(C) Tetrahedral
(D) None of these
21. Which among the following has the largest dipole moment?
(A) NH3
(B) H2O
(C) HI
(D) SO3
22. Amongst H2O, H2S, H2Se and H2Te the one with the highest boiling point is
(a) H2O because of hydrogen bonding
(b) H2Te because of higher molecular weight
(c) H2S because of hydrogen bonding
(d) H2Se because of lower molecular weight.
23.The oxidation number of Cl in Cl2O7 is
(A) + 7
(B) + 5
(C) + 3
(D) – 7
24. The tendency of an electrode to lose electrons is known as
(A) Electrode Potential
(B) Reduction Potential
(C) Oxidation Potential
(D) E.M.F.
25.What takes place in the process of oxidation?
(A) Gain of electrons
(B) Loss of electrons
(C) Both A and B
(D) None of these

SECTION B
This section consists of 24multiple choice questions with overall choice to
attempt any 20 questions. In case more than desirable number of questions are
attempted, ONLY first 20 will be considered for evaluation.
26.Oxidation state of Sulphur in H2SO4 is :
(A) 5
(B) 6
(C)7
(D)8
27.Which of the following hydrides are generally nonstochiometric in nature?
(A) Ionic Hydrides
(B) Molecular Hydrides
(C) Interstitial Hydrides
(D) All Of The Above
28. What is the product of the reaction of H2O2 with Cl2?
(A) O2 + HOCl
(B) HCl + O2
(C) H2O + HCl
(D) HCl + H2
29. Water gas is mixture of hydrogen H2 and
(A) CO
(B) CO2
(C) Cl2
(D) SO2
30.How many istopes does Hydrogen have?
(A) 1
(B) 2
(C) 3
(D) 4
31. If two compounds have the same empirical formula but different molecular
formula they must have
(A) Different Percentage Composition
(B) Different Molecular Weight
(C) Same Viscosity
(D) Same Vapour Density
32. 0.0833mol of carbohydrate of empirical formula CH2O contain 1g of
hydrogen. The molecular formula of the carbohydrate is
(A) C5H10O5
(B) C3H4O3
(C) C12H22O11
(D) C6H12O6
33. Which of the following cannot be represented by resonance structures?
(A) Dimethyl Ether
(B) Nitrate Anion
(C) Carboxylate Anion
(D) Toluene
34. Which one is strongest acid among following options?
(A) CH2FCOOH
(B) CH2ClCOOH
(C) CHCl2COOH
(D) CHF2COOH
35. Find the incorrect statement for a nucleophile
(A) A nucleophile is a Lewis acid
(B) Nucleophiles do not seek electron
(C) Ammonia is a nucleophile
(D) Nucleophiles attack low electron density sites
36. Which among the following is the most deactivating meta–directing group
in aromatic substitution reaction?
(A) –COOH
(B) –SO3H
(C) –NO2
(D) –CN
37. The correct order of increasing nucleophilicity is
(A) Cl– < Br– < I–
(B) Br– <Cl– < I–
(C) I– < Br– <Cl–
(D) I– <Cl– < Br–
38. Among the following alkenes : 1–butane (I), cis–2–butene (II),
trans–2–butene (III), the decreasing order of stability is
(A) III > II > I
(B) III > I > II
(C) I > II > III
(D) II > I > III
39. The displacement of electrons in a multiple bond in the presence of
attacking reagent is called
(A) Inductive effect
(B) Electromeric effect
(C) Resonance
(D) Hyper conjugation
40. Inductive effect involves
(A) displacement of σ electrons
(B) delocalization of π electrons
(C) delocalization of σ–electrons
(D) displacement of π–electrons
41. How many sigma and Pi bonds are there in benzene?
(A) 12 σ , 3 π
(B) 12 σ , 2 π
(C) 12 σ , 4 π
(D) 3 σ , 12 π
42. What is the hybridization of each C atom in Ethane?
(A) sp3, sp3
(B) sp2, sp2
(C) sp3, sp2
(D) sp2, sp3
43. Who discovered Urea molecule?
(A) F Wohler
(B) Berzelius
(C) Dalton
(D) Fridel
44. Paraffins is the name of
(A) Alkane
(B) Alkene
(C) Alkyne
(D) Benzene
Directions: In the following questions, the Assertions (A) and Reason (s) (R)
have been put forward. Read both the statements carefully and choose the
correct alternative from the following:
(A) Both the Assertion and the Reason are correct and the Reason is the
correct explanation of the Assertion.
(B) The Assertion and the Reason are correct but the Reason is not the
correct explanation of the Assertion.
(C) Our Assertion is true but the Reason is false.
(D) The statement of the Assertion is false but the Reason is true.
45.Assertion : Cathode rays originate from cathode and move towards anode.
Reason: Electrons present in the material of cathode are continuously emitted
on applying high voltage.

46.Assertion (A) : All isotopes of a given element show the same type of
chemical
behaviour.
Reason (R) : The chemical properties of an atom are controlled by the
number of electrons in the atom.

47.Assertion (A) : Black body is an ideal body that emits and absorbs
radiations of all frequencies.
Reason (R) : The frequency of radiation emitted by a body goes from a lower
frequency to higher frequency with an increase intemperature.

48.Assertion (A) : It is impossible to determine the exact position and exact


momentum of an electron simultaneously.
Reason (R) : The path of an electron in an atom is clearly defined.

49. Assertion (A) Hydrogen has one electron in its orbit but it produces several
spectral lines.
Reason (R) : There are many excited energy levels available.

Section C
50.Total number SI base units are :
(A) 6
(B) 7
(C)8
(D)9
51.The constant that denotes the number of atoms in 12g of C–12 is called
(A) Avogadro‟s constant
(B) Boltzmann constant
(C)Henry‟s constant
(D)Planck‟s constant
52.If two compounds have the same empirical formula but different molecular
formula they must have
(A) Different Percentage Composition
(B) Different Molecular Weight
(C) Same Viscosity
(D) Same Vapour Density
CASE1: Read the passage given below and answer the following questions 53–
55
Atoms and molecules are extremely small in size and their numbers in even a
small amount of any substance is really very large. To handle such large
numbers, a unit of similar magnitude is required. Just as we denote one dozen
for 12 items, score for 20 items, gross for 144 items, we use the idea of mole to
count entities at the microscopic level (i.e. atoms/molecules/ particles,
electrons, ions, etc). In SI system, mole (symbol, mol) was introduced as
seventh base quantity for the amount of a substance. One mole is the amount
of a substance that contains as many particles or entities as there are atoms in
exactly 12 g (or 0.012 kg) of the 12C isotope. It may be emphasised that the
mole of a substance always contain the same number of entities, no matter
what the substance may be.
53. One mole is the amount of a substance that contains as many particles or
entities as there are atoms in exactly 12 g of :
(A) C–12
(B) C–14
(C) C–13
(D) U–235
54. Number of moles of any substance depends upon :
(A) Chemical nature of substance
(B) Physical nature of substance
(C) Quantity of that substance
(D) None of the above
55. For which of the following, number of moles can be calculated ?
(A) Number of eggs in a basket
(B) Number of cars on the road
(C) Number of atoms in 2g of Hydrogen
(D) All of the above

ANSWER KEY CLASS XI TERM –1


SEC-A ANS SEC–A ANS SEC–B ANS SEC–B ANS SEC –C ANS
1 B 14 A 26 B 39 B 50 B
2 D 15 B 27 C 40 A 51 A
3 B 16 C 28 B 41 A 52 A
4 D 17 B 29 A 42 A 53 A
5 B 18 B 30 C 43 A 54 C
6 D 19 A 31 B 44 A 55 D
7 D 20 A 32 D 45 B
8 D 21 B 33 A 46 A
9 C 22 A 34 A 47 B
10 A 23 A 35 A 48 D
11 C 24 C 36 C 49 A
12 C 25 B 37 A
13 A 38 A
ANIMATED VIDEOS CLASS 11
(SUBJECT: Mathematics in English)

 Chapter :Sequence And Series (Deleted 2023-24) Click here to view animated video
 Chapter :Circle Click here to view animated video
 Chapter :Bionomial theorem Click here to view animated video
 Chapter :Relation And Function Click here to view animated video
 Chapter :Linear Inequalities Click here to view animated video
 Chapter :Quadratic Equation Click here to view animated video
 Chapter :3d Geometry (Deleted 2023-24) Click here to view animated video
 Chapter :Statistics Click here to view animated video
 Chapter :Complex Number Click here to view animated video
 Chapter :Straight Line I Click here to view animated video
 Chapter :Conic Sections Click here to view animated video
 Chapter :Limits And Derivatives Click here to view animated video
 Chapter :Straight Line II Click here to view animated video
 Chapter :Three Dimensional Geometry Click here to view animated video
 Chapter :Linear Equation In One Variable (Deleted 2023-24) Click here to view animated video
 Chapter :Trigonometric Function Click here to view animated video
 Chapter :Mathematical Reasoning (Deleted 2023-24) Click here to view animated video
 Chapter :Vector Algebra(Deleted 2023-24) Click here to view animated video
 Chapter :Permutation And Combination Click here to view animated video
 Chapter :Mathematical Introduction(Deleted 2023-24) Click here to view animated video
 Chapter :Principle Of Mathematical Induction (Deleted 2023-24) Click here to view animated video
 Chapter : Probability Click here to view animated video
 Chapter :Sequence And Series Click here to view animated video
 Chapter :Sets I Click here to view animated video
 Chapter :Sets II Click here to view animated video
 Chapter :Sets III Click here to view animated video
ANIMATED VIDEOS CLASS 11
(SUBJECT:ACCOUNTANCY)

 Chapter : Introduction To Accountancy Click here to view animated video


 Chapter : Accounting System (Deleted 2023-24) Click here to view animated video
 Chapter : Incomplete Record (Deleted 2023-24) Click here to view animated video
 Chapter :Computers In Accounting (Deleted 2023-24) Click here to view animated video
 Chapter : Financial Statement I Click here to view animated video
 Chapter : Financial Statement II Click here to view animated video
 Chapter : Recording Of Transactions I Click here to view animated video
 Chapter : Recording Of Transactions II Click here to view animated video
 Chapter : Structuring Database Of Accounting (Deleted 2023-24) Click here to view animated video
 Chapter : Theory Base Of Accounting Click here to view animated video
 Chapter :Trial Balance And Rectification Of Errors Click here to view animated video

ANIMATED VIDEOS CLASS 11


(SUBJECT: BUSINESS STUDIES)

 Chapter :Business Service Click here to view animated video


 Chapter :Emerging Modes Of Business Click here to view animated video
 Chapter :Formation Of A Company Click here to view animated video
 Chapter :Forms Of Business Organisation Click here to view animated video
 Chapter :Internal Trade Click here to view animated video
 Chapter :International Business Click here to view animated video
 Chapter :Nature And Purpose Of Business (Deleted 2023-24) Click here to view animated video
 Chapter :Private, Public And Global Enterprises Click here to view animated video
 Chapter :Small Business (Deleted 2023-24) Click here to view animated video
 Chapter :Social Responsibilities Of Business Click here to view animated video
And Business Ethics
 Chapter :Source Of Business Finance Click here to view animated video
ANIMATED VIDEOS CLASS 11
(SUBJECT:MICRO ECONOMICS)
 Chapter : Theory Of Consumer Behavior Click here to view animated video
 Chapter : The Theory Of The Firm Under Perfect Competition Click here to view animated video
 Chapter : Production And Costs Click here to view animated video
 Chapter : Non Competitive Markets Click here to view animated video
 Chapter : Market Equilibrium Click here to view animated video
 Chapter : Introduction To Microeconomics Click here to view animated video

ANIMATED VIDEOS CLASS 11


(SUBJECT: STATISTICS)

 Chapter : Use Of Statistical Tools Click here to view animated video


 Chapter : Presentation Of Data Click here to view animated video
 Chapter : Organisation Of Data Click here to view animated video
 Chapter : Measures Of Dispersion (Deleted 2023-24) Click here to view animated video
 Chapter : Measures Of Central Tendency Click here to view animated video
 Chapter : Introduction To Statistics (Deleted 2023-24) Click here to view animated video
 Chapter : Index Numbers Click here to view animated video
 Chapter : Correlation Click here to view animated video
 Chapter : Collection Of Data Click here to view animated video

ANIMATED VIDEOS CLASS 11


(SUBJECT: BIOLOGY IN ENGLISH)
 Chapter :Cell The Unit Of Life Click here to view animated video
 Chapter :Animal Kingdom Click here to view animated video
 Chapter :Mineral Nutrition(Deleted 2023-24) Click here to view animated video
 Chapter :Cell Cycle And Cell Division Click here to view animated video
 Chapter :Anatomy Of Flowering Plants I Click here to view animated video
 Chapter :Respiration In Plants Click here to view animated video
 Chapter :Plant Kingdom II Click here to view animated video
 Chapter :Plant Growth And Development Click here to view animated video
 Chapter :Photosynthesis In Higher Plants Click here to view animated video
 Chapter :Nervous Control And Coordination(Deleted 2023-24) Click here to view animated video

 Chapter :Morphology Of Flowering Plants Click here to view animated video


 Chapter :Morphology Of Flowering Plants Root (Deleted 2023-24) Click here to view animated video
 Chapter :Morphology Of Flowering Plants Leaves(Deleted 2023-24) Click here to view animated video
 Chapter :Transport Of Mineral Nutrients In Plant (Deleted 2023-24) Click here to view animated video
 Chapter :Morphology Of Flowering Plants Inflorescence (Deleted 2023-24) Click here to view animated video
 Chapter :Transport In Plants (Deleted 2023-24) Click here to view animated video
 Chapter :Morphology Of Flowering Plants Fruits Click here to view animated video
 Chapter :The Living Word (Deleted 2023-24) Click here to view animated video
 Chapter :Morphology Of Flowering Plants Flowers (Deleted 2023-24) Click here to view animated video
Chapter :The Living World I Click here to view animated video
 Chapter :Long Distance Transport Of Water (Deleted 2023-24) Click here to view animated video
Chapter : Structural Organisation in Animals Click here to view animated video
 Chapter :Locomotion And Movement Click here to view animated video
 Chapter :Structural Organization In Animals II (Deleted 2023-24) Click here to view animated video
Chapter :Excretory Products And Their Elimination Click here to view animated video
 Chapter :Digestion And Absorption (Deleted 2023-24) Click here to view animated video
 Chapter :Chemical Coordination And Integration Click here to view animated video
 Chapter :Cell The Unit Of Life IV Click here to view animated video
 Chapter :Cell The Unit Of Life III Click here to view animated video
 Chapter :Animal Kingdom II Click here to view animated video
 Chapter :Animal Kingdom III Click here to view animated video
 Chapter :Biological Classification I Click here to view animated video
 Chapter :Biological Classification II Click here to view animated video
 Chapter :Biological Classification III Click here to view animated video
 Chapter : Biomolecules Click here to view animated video
 Chapter :Body Fluids A Circulation Click here to view animated video
 Chapter :Breathing Exchange Of Gases Click here to view animated video
 Chapter :Cell The Unit Of Life I Click here to view animated video
 Chapter :Cell The Unit Of Life II Click here to view animated video
ANIMATED VIDEOS CLASS 11
(SUBJECT: CHEMISTRY IN ENGLISH)

 Chapter :Organic Chemistry III Click here to view animated video


 Chapter :Organic Chemistry II Click here to view animated video
 Chapter :Organic Chemistry I Click here to view animated video
 Chapter :Hydrogen II (Deleted 2023-24) Click here to view animated video
 Chapter :Thermodynamics Click here to view animated video
 Chapter :Hydrogen I (Deleted 2023-24) Click here to view animated video
 Chapter :The S Block Elements (Deleted 2023-24) Click here to view animated video
 Chapter :Hydrocarbons III Click here to view animated video
 Chapter :Hydrocarbons II Click here to view animated video
 Chapter :Structure Of Atoms Click here to view animated video
 Chapter :Hydrocarbons I Click here to view animated video
 Chapter :Structure Of Atoms I Click here to view animated video
 Chapter :Equilibrium Click here to view animated video
 Chapter :States Of Matter (Deleted 2023-24) Click here to view animated video
 Chapter :Equilibrium I Click here to view animated video
 Chapter :Some Basic Concept of Chemistry II Click here to view animated video
 Chapter :Some Basic Concept of Chemistry I Click here to view animated video
 Chapter :Classification Of Elements In And Periodicity Properties Click here to view animated video
 Chapter :Redox Reactions Click here to view animated video
 Chapter :Chemical Bonding Molecular Structure Click here to view animated video
 Chapter :Chemical Bonding Molecular Structure I Click here to view animated video

ANIMATED VIDEOS CLASS 11


(SUBJECT:PHYSICS IN ENGLISH)
 Chapter :Work, Power And Energy Click here to view animated video
 Chapter :Waves Click here to view animated video
 Chapter :Units And Measurement Click here to view animated video
 Chapter :Thermodynamics Click here to view animated video
 Chapter :Thermal Properties Of Matter Click here to view animated video
 Chapter :System Of Particles And Rational Motion Click here to view animated video
 Chapter :Physics World (Deleted 2023-24) Click here to view animated video
 Chapter :Oscillations Click here to view animated video
 Chapter :Motion In A Straight Line Click here to view animated video
 Chapter :Motion In A Plane Click here to view animated video
 Chapter :Mechanical Properties Of Solids Click here to view animated video
 Chapter :Mechanical Properties Of Fluids Click here to view animated video
 Chapter :Laws Of Motion Click here to view animated video
 Chapter :Kinetic Theory Of Gases Click here to view animated video
 Chapter :Gravitation Click here to view animated video

ANIMATED VIDEOS CLASS 12


(SUBJECT: INDIAN ECONOMIC DEVELOPMENT)

 Chapter Indian Economy on the Eve Click here to view animated video
of Independence
 Chapter Indian Economy 1950 1990 Click here to view animated video
Formation in India
 Chapter Development Human Capital Click here to view animated video
 Chapter Environment And Sustainable Click here to view animated video
Development
 Chapter Employment Growth, Informalization Click here to view animated video
And Other Issues
 Chapter Comparative Development Experiences Click here to view animated video
 Chapter Infrastructure Click here to view animated video
 Chapter Liberalization, Privatization Click here to view animated video
or Globalization an Appraisal
 Chapter Poverty Click here to view animated video
 Chapter Rural Development Click here to view animated video
ANIMATED VIDEOS CLASS 11
(SUBJECT: Mathematics in Hindi)

 Chapter :Sequence And Series (Deleted 2023-24) Click here to view animated video
 Chapter :Circle Click here to view animated video
 Chapter :Bionomial theorem Click here to view animated video
 Chapter :Relation And Function Click here to view animated video
 Chapter :Linear Inequalities Click here to view animated video
 Chapter :Quadratic Equation Click here to view animated video
 Chapter :3d Geometry (Deleted 2023-24) Click here to view animated video
 Chapter :Statistics Click here to view animated video
 Chapter :Complex Number Click here to view animated video
 Chapter :Straight Line I Click here to view animated video
 Chapter :Conic Sections Click here to view animated video
 Chapter :Limits And Derivatives Click here to view animated video
 Chapter :Straight Line II Click here to view animated video
 Chapter :Three Dimensional Geometry Click here to view animated video
 Chapter :Linear Equation In One Variable (Deleted 2023-24) Click here to view animated video

 Chapter :Trigonometric Function Click here to view animated video


 Chapter :Mathematical Reasoning (Deleted 2023-24) Click here to view animated video
 Chapter :Vector Algebra(Deleted 2023-24) Click here to view animated video
 Chapter :Permutation And Combination Click here to view animated video
 Chapter :Mathematical Introduction(Deleted 2023-24) Click here to view animated video
 Chapter :Principle Of Mathematical Induction (Deleted 2023-24) Click here to view animated video
 Chapter : Probability Click here to view animated video
 Chapter :Sequence And Series Click here to view animated video
 Chapter :Sets I Click here to view animated video
 Chapter :Sets II Click here to view animated video
 Chapter :Sets III Click here to view animated video
ANIMATED VIDEOS CLASS 11
(SUBJECT: BIOLOGY IN HINDI)
 Chapter :Cell The Unit Of Life Click here to view animated video
 Chapter :Animal Kingdom Click here to view animated video
 Chapter :Mineral Nutrition(Deleted 2023-24) Click here to view animated video
 Chapter :Cell Cycle And Cell Division Click here to view animated video
 Chapter :Anatomy Of Flowering Plants I Click here to view animated video
 Chapter :Respiration In Plants Click here to view animated video
 Chapter :Plant Kingdom II Click here to view animated video
 Chapter :Plant Growth And Development Click here to view animated video
 Chapter :Photosynthesis In Higher Plants Click here to view animated video
 Chapter :Nervous Control And Coordination(Deleted 2023-24) Click here to view animated video

 Chapter :Morphology Of Flowering Plants Click here to view animated video


 Chapter :Morphology Of Flowering Plants Root (Deleted 2023-24) Click here to view animated video
 Chapter :Morphology Of Flowering Plants Leaves(Deleted 2023-24) Click here to view animated video

 Chapter :Transport Of Mineral Nutrients In Plant (Deleted 2023-24) Click here to view animated video

 Chapter :Morphology Of Flowering Plants Inflorescence (Deleted 2023-24) Click here to view animated video
 Chapter :Transport In Plants (Deleted 2023-24) Click here to view animated video
 Chapter :Morphology Of Flowering Plants Fruits Click here to view animated video
 Chapter :The Living Word (Deleted 2023-24) Click here to view animated video
 Chapter :Morphology Of Flowering Plants Flowers (Deleted 2023-24) Click here to view animated video
Chapter :The Living World I Click here to view animated video
 Chapter :Long Distance Transport Of Water (Deleted 2023-24) Click here to view animated video
Chapter : Structural Organisation in Animals Click here to view animated video
 Chapter :Locomotion And Movement Click here to view animated video
 Chapter :Structural Organization In Animals II (Deleted 2023-24) Click here to view animated video
Chapter :Excretory Products And Their Elimination Click here to view animated video
 Chapter :Digestion And Absorption (Deleted 2023-24) Click here to view animated video
 Chapter :Chemical Coordination And Integration Click here to view animated video
 Chapter :Cell The Unit Of Life IV Click here to view animated video
 Chapter :Cell The Unit Of Life III Click here to view animated video
 Chapter :Animal Kingdom II Click here to view animated video
 Chapter :Animal Kingdom III Click here to view animated video
 Chapter :Biological Classification I Click here to view animated video
 Chapter :Biological Classification II Click here to view animated video
 Chapter :Biological Classification III Click here to view animated video
 Chapter : Biomolecules Click here to view animated video
 Chapter :Body Fluids A Circulation Click here to view animated video
 Chapter :Breathing Exchange Of Gases Click here to view animated video
 Chapter :Cell The Unit Of Life I Click here to view animated video
 Chapter :Cell The Unit Of Life II Click here to view animated video

ANIMATED VIDEOS CLASS 11


(SUBJECT:PHYSICS IN HINDI)
 Chapter :Work, Power And Energy Click here to view animated video
 Chapter :Waves Click here to view animated video
 Chapter :Units And Measurement Click here to view animated video
 Chapter :Thermodynamics Click here to view animated video
 Chapter :Thermal Properties Of Matter Click here to view animated video
 Chapter :System Of Particles And Rational Motion Click here to view animated video
 Chapter :Physics World (Deleted 2023-24) Click here to view animated video
 Chapter :Oscillations Click here to view animated video
 Chapter :Motion In A Straight Line Click here to view animated video
 Chapter :Motion In A Plane Click here to view animated video
 Chapter :Mechanical Properties Of Solids Click here to view animated video
 Chapter :Mechanical Properties Of Fluids Click here to view animated video
 Chapter :Laws Of Motion Click here to view animated video
 Chapter :Kinetic Theory Of Gases Click here to view animated video
 Chapter :Gravitation Click here to view animated video

ANIMATED VIDEOS CLASS 11


(SUBJECT: CHEMISTRY IN HINDI)

 Chapter :Organic Chemistry III Click here to view animated video


 Chapter :Organic Chemistry II Click here to view animated video
 Chapter :Organic Chemistry I Click here to view animated video
 Chapter :Hydrogen II (Deleted 2023-24) Click here to view animated video
 Chapter :Thermodynamics Click here to view animated video
 Chapter :Hydrogen I (Deleted 2023-24) Click here to view animated video
 Chapter :The S Block Elements (Deleted 2023-24) Click here to view animated video
 Chapter :Hydrocarbons III Click here to view animated video
 Chapter :Hydrocarbons II Click here to view animated video
 Chapter :Structure Of Atoms Click here to view animated video
 Chapter :Hydrocarbons I Click here to view animated video
 Chapter :Structure Of Atoms I Click here to view animated video
 Chapter :Equilibrium Click here to view animated video
 Chapter :States Of Matter (Deleted 2023-24) Click here to view animated video
 Chapter :Equilibrium I Click here to view animated video
 Chapter :Some Basic Concept of Chemistry II Click here to view animated video
 Chapter :Some Basic Concept of Chemistry I Click here to view animated video
 Chapter :Classification Of Elements In And Periodicity Properties Click here to view animated video
 Chapter :Redox Reactions Click here to view animated video
 Chapter :Chemical Bonding Molecular Structure Click here to view animated video
 Chapter :Chemical Bonding Molecular Structure I Click here to view animated video
Why Artham Resource Material?
Resource materials for teachers and students are essential tools for effective teaching
and learning. They provide valuable information, guidance, and support to both teachers
and students, making the teaching and learning process more efficient and productive.

For teachers, Artham resource materials include lesson plans, instructional guides,
assessment tools, professional development materials, and teaching aids. These
materials are well researched and created according to 2023-24 NEP and NCERT
guidelines.

For students, resource materials can include textbooks, study guides, homework
assignments, reference books, online learning platforms, and educational videos. These
materials can be obtained from school libraries, educational publishers, online
resources, and teachers.

Both teachers and students can also benefit from Artham educational resources which
are free and openly licensed educational materials that can be used and shared for
teaching and learning. Artham resource material include textbooks, courses, lesson
plans, and multimedia resources that are available online.

In summary, resource materials are critical components of effective teaching and


learning. They provide a wealth of information and support that can enhance the quality
of education and help students achieve academic success.

Teachers and students can also purchase these resources from the links provided with
every resource.

JOIN TELEGRAM GROUP/CHANNELS FOR


CLASS WISE HIGH QUALITY RESOURCE
MATERIAL
SOE CBSE Groups
 Click to Join CBSE Group...All classes
 Click to Join SOE CBSE Kindergarten Group
 Click to Join SOE CBSE Class 1 Group
 Click to Join SOE CBSE Class 2 Group
 Click to Join SOE CBSE Class 3 Group
 Click to Join SOE CBSE Class 4 Group
 Click to Join SOE CBSE Class 5 Group
 Click to Join SOE CBSE Class 6 Group
 Click to Join SOE CBSE Class 7 Group
 Click to Join SOE CBSE Class 8 Group
 Click to Join SOE CBSE Class 9 Group
 Click to Join SOE CBSE Class 10 Group
 Click to Join SOE CBSE Class 11 (Science) Group
 Click to Join SOE CBSE Class 11 (Commerce) Group
 Click to Join SOE CBSE Class 11 (Humanities) Group
 Click to Join SOE CBSE Class 12 (Science) Group
 Click to Join SOE CBSE Class 12(Commerce) Group
 Click to Join SOE CBSE Class 12 (Humanities) Group
 Click to Join SOE JEE/NEET Group
 Click to Join SOE CUET Group
 Click to Join SOE NDA, OLYMPIAD, NTSE Group
 Click to Join SOE School Principal Professional Development Group
 Click to Join SOE School Teacher Professional Development Group
 Click to Join SOE CBSE Project File Group for Class 9th to 12th All Subjects

SOE ICSE Groups


 Click to Join SOE ICSE Kindergarten Group
 Click to Join SOE ICSE Class 1 Group
 Click to Join SOE ICSE Class 2 Group
 Click to Join SOE ICSE Class 3 Group
 Click to Join SOE ICSE Class 4 Group
 Click to Join SOE ICSE Class 5 Group
 Click to Join SOE ICSE Class 6 Group
 Click to Join SOE ICSE Class 7 Group
 Click to Join SOE ICSE Class 8 Group
 Click to Join SOE ICSE Class 9 Group
 Click to Join SOE ICSE Class 10 Group
 Click to Join SOE ICSE Class 11 (Science) Group
 Click to Join SOE ICSE Class 11 (Commerce) Group
 Click to Join SOE ICSE Class 11 (Humanities) Group
 Click to Join SOE ICSE Class 12 (Science) Group
 Click to Join SOE ICSE Class 12(Commerce) Group
 Click to Join SOE ICSE Class 12 (Humanities) Group
 Click to Join SOE JEE/NEET Group
 Click to Join SOE CUET Group
 Click to Join SOE NDA, OLYMPIAD, NTSE Group
 Click to Join SOE School Principal Professional Development Group
 Click to Join SOE School Teacher Professional Development Group

Nageen CBSE Channels


 Click to Join Nageen CBSE Kindergarten Channel
 Click to Join Nageen CBSE Class 1 Channel
 Click to Join Nageen CBSE Class 2 Channel
 Click to Join Nageen CBSE Class 3 Channel
 Click to Join Nageen CBSE Class 4 Channel
 Click to Join Nageen CBSE Class 5 Channel
 Click to Join Nageen CBSE Class 6 Channel
 Click to Join Nageen CBSE Class 7 Channel
 Click to Join Nageen CBSE Class 8 Channel
 Click to Join Nageen CBSE Class 9 Channel
 Click to Join Nageen CBSE Class 10 Channel
 Click to Join Nageen CBSE Class 11 (Science) Channel
 Click to Join Nageen CBSE Class 11 (Humanities) Channel
 Click to Join Nageen CBSE Class 11 (Commerce) Channel
 Click to Join Nageen CBSE Class 12 (Science) Channel
 Click to Join Nageen CBSE Class 12 (Commerce) Channel
 Click to Join Nageen CBSE Class 12 (Humanities) Channel
 Click to Join JEE/NEET Channel
 Click to Join CUET Channel
 Click to Join NDA, OLYMPIAD, NTSE Channel

Nageen ICSE Channels


 Click to Join Nageen ICSE Kindergarten Channel
 Click to Join Nageen ICSE Class 1 Channel
 Click to Join Nageen ICSE Class 2 Channel
 Click to Join Nageen ICSE Class 3 Channel
 Click to Join Nageen ICSE Class 4 Channel
 Click to Join Nageen ICSE Class 5 Channel
 Click to Join Nageen ICSE Class 6 Channel
 Click to Join Nageen ICSE Class 7 Channel
 Click to Join Nageen ICSE Class 8 Channel
 Click to Join Nageen ICSE Class 9 Channel
 Click to Join Nageen ICSE Class 10 Channel
 Click to Join Nageen ICSE Class 11 (Science) Channel
 Click to Join Nageen ICSE Class 11 (Commerce) Channel
 Click to Join Nageen ICSE Class 11 (Humanities) Channel
 Click to Join Nageen ICSE Class 12 (Science) Channel
 Click to Join Nageen ICSE Class 12 (Commerce) Channel
 Click to Join Nageen ICSE Class 12 (Humanities) Channel
 Click to Join JEE/NEET Channel
 Click to Join CUET Channel
 Click to Join NDA, OLYMPIAD, NTSE Channel
SCHOOL OF EDUCATORS
You will get Pre- Board Papers PDF, Word file, PPT, Lesson Plan, Worksheet, practical tips and Viva
questions , reference books , smart content , curriculum , syllabus , marking scheme , toppers answer
scripts , revised exam pattern , revised syllabus , Blue Print etc. here .Join Your Subject WhatsApp
Group.

Kindergarten

Class 1 Class 2 Class 3 Class 4

Class 5 Class 6 Class 7 Class 8

Class 9 Class 10 Class 11 (Science) Class 11 (Commerce)

Class 11 (Humanities) Class 12 (Science) Class 12 (Commerce) Class 12 (Humanities)


Subject Wise Groups Secondary and Senior Secondary

Secondary Groups (IX & X)


SST Mathematics Science English

Hindi Information Techonology (402)

Senior Secondary Groups (XI & XII)


Physics Chemistry English Mathematics

Biology Accountancy Economics BST

History Geography Sociology Hindi Elective

Hindi Core Home Science Sanskrit Psychology

Political Science Painting Vocal Music Comp. Science

IP Physical Education App. Mathematics IIT /NEET

Leagal Studies Entrepreneurship French Teachers Jobs

SOE CBSE Principals (Group for Principals Only)


Rules & Regulations of the Group

1. No introduction
2. No Good Morning/Any wish type message
3.No personal Chats & Messages
4. No Spam
5. You can also ask your difficulties here.

Just get learning resources & post learning resources.


Helpline number only WhatsApp: +91-95208-77777
n o u r
Joi G r o up
g r a m
Tele s a m p le
o r e
for m
a p e r s
p

Join o
u
Teleg r
ra
for m m Chann
ore b el
Notes o oks &

Pdf Purchase
Print Purchase Scan QR Code to join
Telegram Groups &
Channels for more free
Amazon Print Purchase resources.

You might also like